Как составлять уравнения равновесия в теоретической механике

iSopromat.ru

Уравнения равновесия (статики) характеризуют неподвижность заданной системы нагруженной комплексом внешних усилий.

При решении задач теоретической механики и сопротивления материалов (например, при определении опорных реакций или внутренних силовых факторов) исходя из условия неподвижности системы или ее частей, записываются уравнения равенства нулю сумм проекций всех сил на оси выбранной системы координат

что следует из условия отсутствия перемещения системы вдоль этих осей, и сумм моментов относительно произвольных точек системы

из условия отсутствия ее вращения относительно указанных осей.

Надо отметить что в случае действия плоской системы сил можно получить только три уравнения статики, а линейная схема нагружения позволяет записать лишь одно уравнение.

Пример составления уравнений равновесия

В качестве примера, рассмотрим общий случай пространственного нагружения, где комплекс усилий, включающий сосредоточенные силы F1-F6, равномерно распределенную нагрузку q, и момент m расположенный в плоскости перпендикулярной длинному стержню, удерживает L-образную систему в равновесии.

Обозначим характерные точки системы буквами A, B, C и D, зададим положение трехмерной системы координат xyz и запишем уравнения равновесия.

Суммы проекций сил

Сумма проекций всех сил на ось x (с учетом правила знаков для сил):

здесь при записи силы от распределенной нагрузки ее интенсивность q умножается на ее длину AB.

Суммы моментов

Суммы моментов всех нагрузок, например, относительно точки B (с учетом правила знаков для моментов):

  • в плоскости xOy:
  • в плоскости xOz:
  • в плоскости yOz:

Из полученных шести уравнений можно определить не более шести неизвестных усилий.

Уважаемые студенты!
На нашем сайте можно получить помощь по техническим и другим предметам:
✔ Решение задач и контрольных
✔ Выполнение учебных работ
✔ Помощь на экзаменах

Теоретическая механика. В помощь студенту

Теоретическая механика – это раздел механики, в котором излагаются основные законы механического движения и механического взаимодействия материальных тел.

Теоретическая механика является наукой, в которой изучаются перемещения тел с течением времени (механические движения). Она служит базой других разделов механики (теория упругости, сопротивление материалов, теория пластичности, теория механизмов и машин, гидроаэродинамика) и многих технических дисциплин.

Механическое движение — это изменение с течением времени взаимного положения в пространстве материальных тел.

Механическое взаимодействие – это такое взаимодействие, в результате которого изменяется механическое движение или изменяется взаимное положение частей тела.

Статика твердого тела

Статика — это раздел теоретической механики, в котором рассматриваются задачи на равновесие твердых тел и преобразования одной системы сил в другую, ей эквивалентную.

    Основные понятия и законы статики

  • Абсолютно твердое тело (твердое тело, тело) – это материальное тело, расстояние между любыми точками в котором не изменяется.
  • Материальная точка – это тело, размерами которого по условиям задачи можно пренебречь.
  • Свободное тело – это тело, на перемещение которого не наложено никаких ограничений.
  • Несвободное (связанное) тело – это тело, на перемещение которого наложены ограничения.
  • Связи – это тела, препятствующие перемещению рассматриваемого объекта (тела или системы тел).
  • Реакция связи — это сила, характеризующая действие связи на твердое тело. Если считать силу, с которой твердое тело действует на связь, действием, то реакция связи является противодействием. При этом сила — действие приложена к связи, а реакция связи приложена к твердому телу.
  • Механическая система – это совокупность взаимосвязанных между собой тел или материальных точек.
  • Твердое тело можно рассматривать как механическую систему, положения и расстояние между точками которой не изменяются.
  • Сила – это векторная величина, характеризующая механическое действие одного материального тела на другое.
    Сила как вектор характеризуется точкой приложения, направлением действия и абсолютным значением. Единица измерения модуля силы – Ньютон.
  • Линия действия силы – это прямая, вдоль которой направлен вектор силы.
  • Сосредоточенная сила – сила, приложенная в одной точке.
  • Распределенные силы (распределенная нагрузка) – это силы, действующие на все точки объема, поверхности или длины тела.
    Распределенная нагрузка задается силой, действующей на единицу объема (поверхности, длины).
    Размерность распределенной нагрузки – Н/м 3 (Н/м 2 , Н/м).
  • Внешняя сила – это сила, действующая со стороны тела, не принадлежащего рассматриваемой механической системе.
  • Внутренняя сила – это сила, действующая на материальную точку механической системы со стороны другой материальной точки, принадлежащей рассматриваемой системе.
  • Система сил – это совокупность сил, действующих на механическую систему.
  • Плоская система сил – это система сил, линии действия которых лежат в одной плоскости.
  • Пространственная система сил – это система сил, линии действия которых не лежат в одной плоскости.
  • Система сходящихся сил – это система сил, линии действия которых пересекаются в одной точке.
  • Произвольная система сил – это система сил, линии действия которых не пересекаются в одной точке.
  • Эквивалентные системы сил – это такие системы сил, замена которых одна на другую не изменяет механического состояния тела.
    Принятое обозначение: .
  • Равновесие – это состояние, при котором тело при действии сил остается неподвижным или движется равномерно прямолинейно.
  • Уравновешенная система сил – это система сил, которая будучи приложена к свободному твердому телу не изменяет его механического состояния (не выводит из равновесия).
    .
  • Равнодействующая сила – это сила, действие которой на тело эквивалентно действию системы сил.
    .
  • Момент силы – это величина, характеризующая вращающую способность силы.
  • Пара сил – это система двух параллельных равных по модулю противоположно направленных сил.
    Принятое обозначение: .
    Под действием пары сил тело будет совершать вращательное движение.
  • Проекция силы на ось – это отрезок, заключенный между перпендикулярами, проведенными из начала и конца вектора силы к этой оси.
    Проекция положительна, если направление отрезка совпадает с положительным направлением оси.
  • Проекция силы на плоскость – это вектор на плоскости, заключенный между перпендикулярами, проведенными из начала и конца вектора силы к этой плоскости.
  • Закон 1 (закон инерции). Изолированная материальная точка находится в покое либо движется равномерно и прямолинейно.
    Равномерное и прямолинейное движение материальной точки является движением по инерции. Под состоянием равновесия материальной точки и твердого тела понимают не только состояние покоя, но и движение по инерции. Для твердого тела существуют различные виды движения по инерции, например равномерное вращение твердого тела вокруг неподвижной оси.
  • Закон 2. Твердое тело находится в равновесии под действием двух сил только в том случае, если эти силы равны по модулю и направлены в противоположные стороны по общей линии действия.
    Эти две силы называются уравновешивающимися.
    Вообще силы называются уравновешивающимися, если твердое тело, к которому приложены эти силы, находится в покое.
  • Закон 3. Не нарушая состояния (слово «состояние» здесь означает состояние движения или покоя) твердого тела, можно добавлять и отбрасывать уравновешивающиеся силы.
    Следствие. Не нарушая состояния твердого тела, силу можно переносить по ее линии действия в любую точку тела.
    Две системы сил называются эквивалентными, если одну из них можно заменить другой, не нарушая состояния твердого тела.
  • Закон 4. Равнодействующая двух сил, приложенных в одной точке, приложена в той же точке, равна по модулю диагонали параллелограмма, построенного на этих силах, и направлена вдоль этой
    диагонали.
    По модулю равнодействующая равна:
  • Закон 5 (закон равенства действия и противодействия). Силы, с которыми два тела действуют друг на друга, равны по модулю и направлены в противоположные стороны по одной прямой.
    Следует иметь в виду, что действие — сила, приложенная к телу Б, и противодействие — сила, приложенная к телу А, не уравновешиваются, так как они приложены к разным телам.
  • Закон 6 (закон отвердевания). Равновесие нетвердого тела не нарушается при его затвердевании.
    Не следует при этом забывать, что условия равновесия, являющиеся необходимыми и достаточными для твердого тела, являются необходимыми, но недостаточными для соответствующего нетвердого тела.
  • Закон 7 (закон освобождаемости от связей). Несвободное твердое тело можно рассматривать как свободное, если его мысленно освободить от связей, заменив действие связей соответствующими реакциями связей.
    • Связи и их реакции

    • Гладкая поверхность ограничивает перемещение по нормали к поверхности опоры. Реакция направлена перпендикулярно поверхности.
    • Шарнирная подвижная опора ограничивает перемещение тела по нормали к опорной плоскости. Реакция направлена по нормали к поверхности опоры.
    • Шарнирная неподвижная опора противодействует любому перемещению в плоскости, перпендикулярной оси вращения.
    • Шарнирный невесомый стержень противодействует перемещению тела вдоль линии стержня. Реакция будет направлена вдоль линии стержня.
    • Глухая заделка противодействует любому перемещению и вращению в плоскости. Ее действие можно заменить силой, представленной в виде двух составляющих и парой сил с моментом.
      Момент силы относительно точки

    • Абсолютное значение момента равно произведению модуля силы на кратчайшее расстояние h от центра вращения до линии действия силы. Расстояние h называют плечом силы.
    • Момент считают положительным, если сила стремится вращать плечо h против хода часовой стрелки и отрицательным при вращении по ходу часовой стрелки.
    • Свойства момента силы относительно точки:
      1) Момент силы не изменится при переносе точки приложения силы вдоль линии действия силы.
      2) Момент силы равен нулю, если линия действия силы проходит через точку приложения силы.
      3) Момент равнодействующей силы относительно точки равен сумме моментов слагаемых сил относительно этой точки.
      ,
      где
      Момент силы относительно оси

    • Момент силы относительно оси — это момент проекции этой силы на плоскость, перпендикулярную оси, относительно точки пересечения оси с плоскостью.
      Момент считается положительным, если с положительного конца оси поворот, который сила стремится совершить, виден происходящим против хода часовой стрелки, и отрицательным – если по ходу часовой стрелки.
    • Чтобы найти момент силы относительно оси, нужно:
      1) Провести плоскость перпендикулярную оси z.
      2) Спроецировать силу на эту плоскость и вычислить величину проекции .
      3) Провести плечо h из точки пересечения оси с плоскостью на линию действия проекции силы и вычислить его длину.
      4) Найти произведение этого плеча и проекции силы с соответствующим знаком.
    • Свойства момента силы относительно оси.
      Момент силы относительно оси равен нулю, если:
      1) , то есть сила параллельна оси.
      2) h=0, то есть линия действия силы пересекает ось.
      Момент пары сил

    • Момент пары сил равен произведению одной силы на кратчайшее расстояние между линиями действия сил пары, которое называется плечом пары (пара сил оказывает на тело вращающее действие)
      ,
      где: — силы, составляющие пару;
      h — плечо пары.
      Момент пары считают положительным, если силы стремятся вращать плечо против хода часовой стрелки.
    • Свойства пары сил.
      1) Сумма проекций сил пары на любую ось равна нулю.
      2) Не изменяя момента пары можно одновременно соответственно изменять значение сил и плечо пары.
      3) Пару можно переносить в плоскости ее действия при этом действие пары на тело не изменится.
      Преобразование сходящейся системы сил

    • Равнодействующая двух сходящихся сил находится на основании аксиомы о параллелограмме сил.
      Геометрическая сумма любого числа сходящихся сил может быть определена путем последовательного сложения двух сил – способ векторного многоугольника.
      Вывод: система сходящихся сил () приводится к одной равнодействующей силе .
    • Аналитически равнодействующая сила может быть определена через ее проекции на оси координат:

      Согласно теореме: проекция равнодействующей на ось равна сумме проекций слагаемых сил на эту ось: , или в общем виде
      С учетом равнодействующая определяется выражением:
      .
    • Направление вектора равнодействующей определяется косинусами углов между вектором и осями x, y, z:
      Преобразование произвольной системы сил

    • Теорема: силу, приложенную к твердому телу, можно, не изменяя оказываемого ею действия, перенести параллельно в другую точку тела, прибавляя при этом пару сил с моментом, равным моменту переносимой силы относительно точки, в которую она переносится.
      В результате указанного преобразования получается сходящаяся система сил и сумма моментов пар сил. Действие сходящейся системы сил заменяют действием суммарной силы, действие моментов — суммарным моментом.
      Суммарный вектор — это главный вектор системы сил.
      Суммарный момент — это главный момент системы сил.
      Вывод: произвольная система сил в результате тождественного преобразования приводится к главному вектору и главному моменту системы сил.
    • Аналитически главный вектор и главный момент системы сил могут быть определены через их проекции на оси координат:
      ,
      Условия равновесия систем сил

    • Равновесие системы сходящихся сил
      Действие системы сходящихся сил эквивалентно действию одной равнодействующей силы.
      Для равновесия тела необходимо и достаточно, чтобы равнодействующая равнялась нулю .
      Из формулы следует, что для равновесия пространственной системы сходящихся сил необходимо и достаточно, чтобы сумма проекций всех сил на оси X,Y,Z равнялась нулю:
    • Для равновесия плоской сходящейся системы сил необходимо и достаточно, чтобы сумма проекций всех сил на оси X,Y равнялась нулю:
      Равновесие произвольной системы сил.

    • Действие произвольной системы сил эквивалентно действию главного вектора и главного момента. Для равновесия необходимо и достаточно выполнения условия:
      .
    • Для равновесия произвольной системы сил необходимо и достаточно, чтобы суммы проекций всех сил на оси X,Y,Z и суммы моментов всех сил относительно осей X,Y,Z равнялись нулю:
    • Для равновесия плоской произвольной системы сил необходимо и достаточно, чтобы сумма проекций главного вектора на оси X,Y, и алгебраическая сумма моментов сил относительно центра О были равны нулю:

    Кинематика

    Кинематика — раздел теоретической механики, в котором рассматриваются общие геометрические свойства механического движения, как процесса, происходящего в пространстве и во времени. Движущиеся объекты рассматривают как геометрические точки или геометрические тела.

      Основные понятия кинематики

  • Закон движения точки (тела) – это зависимость положения точки (тела) в пространстве от времени.
  • Траектория точки – это геометрическое место положений точки в пространстве при ее движении.
  • Скорость точки (тела) – это характеристика изменения во времени положения точки (тела) в пространстве.
  • Ускорение точки (тела) – это характеристика изменения во времени скорости точки (тела).
    • Способы задания движения точки

    • Задать движение точки — значит задать изменение ее положения по отношению к выбранной системе отсчета. Существуют три основные системы отсчета: векторная, координатная, естественная.
    • В векторной системе положение точки относительно начала отсчета задается радиус-вектором.
      Закон движения: .
    • В системе координат OXYZ положение точки задается тремя координатами X, Y, Z.
      Закон движения: x = x(t), y = y(t); z = z(t).
    • В естественной системе отсчета положение точки задается расстоянием S от начала отсчета до этой точки вдоль траектории.
      Закон движения: .
      Движение точки, при естественном способе задания движения, определено если известны:
      1) Траектория движения.
      2) Начало и направление отсчета дуговой координаты.
      3) Уравнение движения.
      При естественном способе задания движения, в отличии от других способов, используются подвижные координатные оси, движущиеся вместе с точкой по траектории. Такими осями являются:
      Касательная (τ) – направлена в сторону возрастания дуговой координаты по касательной к траектории.
      Главная нормаль (n) – направлена в сторону вогнутости кривой.
      Бинормаль (b) – направлена перпендикулярно к осям τ, n.
      Определение кинематических характеристик точки

    • Траектория точки
      В векторной системе отсчета траектория описывается выражением: .
      В координатной системе отсчета траектория определяется по закону движения точки и описывается выражениями z = f(x,y) — в пространстве, или y = f(x) – в плоскости.
      В естественной системе отсчета траектория задается заранее.
    • Определение скорости точки в векторной системе координат
      При задании движения точки в векторной системе координат отношение перемещения к интервалу времени называют средним значением скорости на этом интервале времени: .
      Принимая интервал времени бесконечно малой величиной, получают значение скорости в данный момент времени (мгновенное значение скорости): .
      Вектор средней скорости направлен вдоль вектора в сторону движения точки, вектор мгновенной скорости направлен по касательной к траектории в сторону движения точки.
      Вывод:скорость точки – векторная величина, равная производной от закона движения по времени.
      Свойство производной:производная от какой либо величины по времени определяет скорость изменения этой величины.
    • Определение скорости точки в координатной системе отсчета
      Скорости изменения координат точки:
      .
      Модуль полной скорости точки при прямоугольной системе координат будет равен:
      .
      Направление вектора скорости определяется косинусами направляющих углов:
      ,
      где — углы между вектором скорости и осями координат.
    • Определение скорости точки в естественной системе отсчета
      Скорость точки в естественной системе отсчета определяется как производная от закона движения точки: .
      Согласно предыдущим выводам вектор скорости направлен по касательной к траектории в сторону движения точки и в осях определяется только одной проекцией .
      Ускорение точки

    • По определению ускорение характеризует изменение скорости, то есть скорость изменения скорости.
    • Ускорения точки в векторной системе отсчета
      На основании свойства производной:
      .
      Вектор скорости может изменяться по модулю и направлению.
      Вектор ускорения направлен по линии приращения вектора скорости, т. е. в сторону искривления траектории.
    • Ускорение точки в координатной системе отсчета
      Ускорение изменения координат точки равно производной по времени от скоростей изменения этих координат:
      .
      Полное ускорение в прямоугольной системе координат будет определяться выражением:
      .
      Направляющие косинусы вектора ускорения:
      .
    • Ускорение точки в естественной системе отсчета Приращение вектора скорости можно разложить на составляющие, параллельные осям естественной системы координат:
      .
      Разделив левую и правую части равенства на dt, получим:
      ,
      где — тангенциальное ускорение;
      — нормальное ускорение;
      R — радиус кривизны траектории в окрестности точки.
      Кинематика твердого тела

    • В кинематике твердых тел решаются две основные задачи:
      1) задание движения и определение кинематических характеристик тела в целом;
      2) определение кинематических характеристик точек тела.
    • Поступательное движение твердого тела
      Поступательное движение — это движение, при котором прямая, проведенная через две точки тела, остается параллельной ее первоначальному положению.
      Теорема:при поступательном движении все точки тела движутся по одинаковым траекториям и имеют в каждой момент времени одинаковые по модулю и направлению скорости и ускорения.
      Вывод:поступательное движение твердого тела определяется движением любой его точки, в связи с чем, задание и изучение его движения сводится к кинематике точки.
    • Вращательное движение твердого тела вокруг неподвижной оси
      Вращательное движение твердого тела вокруг неподвижной оси — это движение твердого тела, при котором две точки, принадлежащие телу, остаются неподвижными в течение всего времени движения.
      Положение тела определяется углом поворота . Единица измерения угла – радиан. (Радиан — центральный угол окружности, длина дуги которого равна радиусу, полный угол окружности содержит радиана.)
      Закон вращательного движения тела вокруг неподвижной оси .
      Угловую скорость и угловое ускорение тела определим методом дифференцирования:
      — угловая скорость, рад/с;
      — угловое ускорение, рад/с².
      Если рассечь тело плоскостью перпендикулярной оси, выбрать на оси вращения точку С и произвольную точку М, то точка М будет описывать вокруг точки С окружность радиуса R. За время dt происходит элементарный поворот на угол , при этом точка М совершит перемещение вдоль траектории на расстояние .
      Модуль линейной скорости:
      .
      Ускорение точки М при известной траектории определяется по его составляющим :
      ,
      где .
      В итоге, получаем формулы
      тангенциальное ускорение: ;
      нормальное ускорение: .
      Плоско-параллельное движение твердого тела

    • Плоско-параллельное движение твердого тела — это движение твердого тела, при котором все его точки перемещаются в плоскостях, параллельных одной неподвижной плоскости.
      Движение сечения S в своей плоскости можно рассматривать как сложное, состоящее из двух элементарных движений:
      1) поступательного и вращательного;
      2) вращательного относительно подвижного (мгновенного) центра.
    • В первом варианте движение сечения может быть задано уравнениями движения одной его точки (полюса) и вращением сечения вокруг полюса.
      В качестве полюса может быть принята любая точка сечения.
      Уравнения движения запишутся в виде:
      .
      Ускорение точки движущейся плоской фигуры складывается из ускорения полюса относительно неподвижной системы отсчета и ускорения за счет вращательного движения вокруг полюса.

    • Во втором варианте движение сечения рассматривается как вращательное вокруг подвижного (мгновенного) центра P.
      В этом случае скорость любой точки В сечения будет определяться по формуле для вращательного движения:
      .
      Угловая скорость вокруг мгновенного центра Р может быть определена если известна скорость какой либо точки сечения, например точки А.
      .
    • Положение мгновенного центра вращения может быть определено на основании следующих свойств:
      1) вектор скорости точки перпендикулярен радиусу;
      2) модуль скорости точки пропорционален расстоянию от точки до центра вращения ();
      3) скорость в центре вращения равна нулю.
    • Теорема:проекции скоростей двух точек твердого тела на прямую, проведенную через эти точки, равны между собой и одинаково направлены.
      Доказательство: расстояние АВ изменяться не может, следовательно, не может быть больше или меньше .
      Вывод:.
      Сложное движение точки

    • Относительное движение — это движение точки относительно подвижной системы.
      Переносное движение — это движение точки вместе с подвижной системой.
      Абсолютное движение — это движение точки относительно неподвижной системы.
      Соответственно называют скорости и ускорения:
      — относительные;
      — переносные;
      — абсолютные.
    • Абсолютная скорость точки равна векторной сумме относительной и переносной скоростей (согласно теореме о сложении скоростей):
      .
      Абсолютное значение скорости определяется по теореме косинусов:
      .
    • Ускорение по правилу параллелограмма определяется только при поступательном переносном движении
      .
      .
    • При непоступательном переносном движении появляется третья составляющая ускорения, называемое поворотным или кориолисовым.
      ,
      где .
      Кориолисово ускорение численно равно:
      ,
      где – угол между векторами и .
      Направление вектора кориолисова ускорения удобно определять по правилу Н.Е. Жуковского: вектор спроектировать на плоскость, перпендикулярную оси переносного вращения, проекцию повернуть на 90 градусов в сторону переносного вращения. Полученное направление будет соответствовать направлению кориолисова ускорения.

    Динамика

    Динамика — это раздел теоретической механики, в котором изучаются механические движении материальных тел в зависимости от причин, их вызывающих.

      Основные понятия динамики

  • Инерционность — это свойство материальных тел сохранять состояние покоя или равномерного прямолинейного движения, пока внешние силы не изменят этого состояния.
  • Масса — это количественная мера инерционности тела. Единица измерения массы — килограмм (кг).
  • Материальная точка — это тело, обладающее массой, размерами которого при решении данной задачи пренебрегают.
  • Центр масс механической системы — геометрическая точка, координаты которой определяются формулами:

    где mk, xk, yk, zk — масса и координаты k-той точки механической системы, m — масса системы.
    В однородном поле тяжести положение центра масс совпадает с положением центра тяжести.
  • Момент инерции материального тела относительно оси – это количественная мера инертности при вращательном движении.
    Момент инерции материальной точки относительно оси равен произведению массы точки на квадрат расстояния точки от оси:
    .
    Момент инерции системы (тела) относительно оси равен арифметической сумме моментов инерции всех точек:
  • Сила инерции материальной точки — это векторная величина, равная по модулю произведению массы точки на модуль ускорения и направленная противоположно вектору ускорения:
  • Сила инерции материального тела — это векторная величина, равная по модулю произведению массы тела на модуль ускорения центра масс тела и направленная противоположно вектору ускорения центра масс: ,
    где — ускорение центра масс тела.
  • Элементарный импульс силы — это векторная величина , равная произведению вектора силы на бесконечно малый промежуток времени dt:
    .
    Полный импульс силы за Δt равен интегралу от элементарных импульсов:
    .
  • Элементарная работа силы — это скалярная величина dA, равная скалярному произведению вектора силы на бесконечно малое перемещение .
    Скалярное произведение векторов равно произведению их модулей на косинус угла между направлениями векторов:
    ,
    где α — угол между направлениями векторов перемещения и силы.
  • Работа силы на конечном перемещении точки её приложения равна интегралу от элементарной работы, взятому по перемещению:
    .
    Единица измерения работы — Джоуль (1 Дж = 1 Н·м).
  • Количество движения материальной точки — это векторная величина , равная произведению массы m на её скорость :
    .
  • Количество движения механической системы равно векторной сумме количества движения её точек.
    или
    ,
    где m — масса механической системы, — вектор скорости центра масс системы.
  • Кинетическая энергия материальной точки — это скалярная величина Т, равная половине произведения массы точки на квадрат её скорости:
    .
  • Кинетическая энергия механической системы равна сумме кинетических энергий всех её точек:
    .
    • Аксиомы динамики

    • Первая аксиома — это закон инерции.
      Если на свободную материальную точку не действуют никакие силы или действует уравновешенная система сил, то точка будет находиться в состоянии покоя или равномерного прямолинейного движения.
    • Вторая аксиома — закон пропорциональности ускорения.
      Ускорение, сообщаемое материальной точке действующей на неё силой, пропорционально этой силе и по направлению совпадает с направлением силы: — это основной закон динамики.
    • Третья аксиома — это закон противодействия.
      Силы, с которыми действуют друг на друга две материальные точки, равны по модулю и направлены вдоль прямой, соединяющей эти точки, в противоположные стороны:
      .
    • Четвертая аксиома — закон независимости действия сил.
      При действии на материальную точку системы сил полное ускорение этой точки равно геометрической сумме ускорений от действия каждой силы:
      Дифференциальные уравнения динамики

    • Дифференциальные уравнения движения точки связывают ускорение точки с действующими на нее силами. Фактически дифференциальные уравнения являются записью основного закона динамики в явной дифференциальной форме.
      Для абсолютного движения точки (движение в инерциальной системе отсчета) дифференциальное уравнение имеет вид:
      .
    • Векторное уравнение может быть записано в проекциях на оси прямоугольной инерциальной системы координат:
    • При известной траектория движения точки уравнение может быть записано в проекциях на оси естественной системы координат:

      С учетом того, что ,
      где — тангенциальное ускорение;
      — нормальное ускорение,
      уравнения примут вид:
      Общие теоремы динамики

    • Общие теоремы динамики устанавливают зависимость между мерами механического движения и механического взаимодействия. Выводы теорем являются результатом тождественного преобразования основного закона динамики.
    • Теорема об изменении количества движения: изменение количества движения материальной точки (механической системы) за конечный промежуток времени равно сумме импульсов внешних сил за тот же промежуток времени — для материальной точки;
      — для механической системы.
    • Теорема об изменении кинетической энергии: изменение кинетической энергии точки (механической системы) при её перемещении равно сумме работ всех действующих внешних сил на этом перемещении — для материальной точки;
      — для механической системы.
    • Кинетическая энергия механической системы определяется в соответствии с , при этом для твердых тел выведены следующие зависимости:
      — при поступательном движении тела;
      — при вращательном движении тела;
      — при плоско-параллельном движении тела.
    • Момент инерции цилиндра относительно его оси:
      .
    • Момент инерции стержня относительно оси z:
      .
    • Момент инерции прямоугольной пластины относительно осей х и y: .
    • Момент инерции шара определяется по формуле:
      .
    • Работа силы тяжести:
      ,
      где P — сила тяжести;
      h — изменение положения тела по вертикали.
    • Работа силы при вращательном движении тела
      ,
      где M — момент силы,
      w — угловая скорость тела.
      Следует иметь в виду, что работа, как скалярная величина, может быть положительной или отрицательной. Работа будет положительной если направление действия силы совпадает с направлением движения.
      Принцип Даламбера

    • Формулировка принципа Даламбера: если в любой момент времени к действующим на точку силам присоединить силы инерции, то полученная система сил будет уравновешенной:
      .
    • Для механической системы:
      .

    Примеры решения задач

    Решение примеров по теме: «Статика твердого тела»

    Пример 1. Условия равновесия


    Висящий на нити, под углом в сорок пять градусов к гладкой стене шар весом в десять Ньютон, находится в состоянии равновесия (рис. а). Необходимо определить давление однородного шара на гладкую стенку и натяжение нити.

    Дано: P = 10 Н; α = 45°
    Найти: N, T — ?

    Решение.
    Отбрасываем связи, а их действие на шар заменяем реакциями.
    Реакция стенки N направлена перпендикулярно стенке (от точки касания С к центру шара О), реакция нити Т — вдоль нити от точки А к точке В.
    Тем самым выявляется полная система сил, приложенных к покоящемуся шару.

    Это система сил, сходящихся в центре О шара, и состоящая из веса шара Р (активная сила), реакции стенки N и реакции нити Т (рис. б).

    Реакции N и Т по величине неизвестны. Для их определения следует воспользоваться условиями равновесия (в той или иной форме — геометрической, аналитической).

    При геометрическом способе решения строится замкнутый многоугольник сил и используются соотношения школьной геометрии (теорема синусов, теорема косинусов, теорема Пифагора и т.д.).

    В данном случае это замкнутый силовой треугольник (рис. в), из которого получаем:

    После подстановки в формулы числовых значений, получим:
    .

    Ответ: .

    Решение примеров по теме: «Кинематика»

    Пример 2. Уравнение траектории точки

    Дано:
    Движение точки задано уравнениями ;
    (x, у — в сантиметрах, t — в секундах).
    Найти: уравнение траектории точки в координатной форме.

    Решение. Для определения уравнения траектории из уравнений движения исключаем время t. Для этого из первого уравнения выражаем и подставляем это значение во второе уравнение, преобразованное к функциям одинарного угла:
    .

    Опуская промежуточные выражения, получаем уравнение траектории:
    .

    Уравнение определяет параболу, расположенную симметрично относительно оси у, с вершиной в точке (0, 4). Траекторией служит кусок этой параболы, заключенный между точками с координатами (-2, -4) и (2, -4).

    Ответ: .

    Решение примеров по теме: «Динамика»

    Пример 3. Основной закон динамики точки

    Свободная материальная точка, масса которой десять килограмм, движется прямолинейно с ускорением пол метра в секунду в квадрате. Определить силу, приложенную к точке.

    Дано: m = 10 кг; a = 0,5 м/с 2 .
    Найти: F — ?

    Решение.
    Согласно основному закону динамики: .

    Подставив значения в формулу, получим:

    Ответ: сила, сообщающая массе, равной 10 кг,
    ускорение 0,5 м/с 2 , равна 5 Н.

    В помощь студенту
      Формулы, правила, законы, теоремы, уравнения, примеры решения задач

    Список литературы:
    Бать М.И., Джанелидзе Г.Ю., Кельзон А.С. Теоретическая механика в примерах и задачах.
    Буторин Л.В., Бусыгина Е.Б. Теоретическая механика. Учебно-практическое пособие.

    Плоская система сил в теоретической механике

    Содержание:

    Плоская система сил:

    Плоскую систему сил можно привести к более простой системе сил, состоящей из силы или пары сил. Эти случаи возможны, если система сил не находится в равновесии, т. е. если одновременно не равны нулю главные вектор и момент системы сил. Рассмотрим эти частные случаи.

    Случай приведения к равнодействующей силе

    1. Если при приведении плоской системы сил к какому-либо центру окажется, что главный вектор Равнодействующая сила в этом случае проходит через центр приведения, а по величине и направлению совпадает с главным вектором .
    2. Если при приведении плоской системы сил главный вектор и главный момент , то такую систему можно упростить и привести к одной равнодействующей силе .

    Эта сила по величине и направлению совпадает с главным вектором , но ее линия действия отстоит от первоначального центра приведения на расстоянии (рис. 40), которое определяют из соотношения

    Рис. 40

    Действительно, пусть при приведении к точке получаются главный вектор и пара сил, алгебраический момент которой равен главному моменту . По теореме об эквивалентности пар сил, расположенных в одной плоскости, пару сил можно поворачивать, передвигать в плоскости ее действия и изменять плечо и силы пары, сохраняя ее алгебраический момент. Выберем силы , , входящие в пару сил, равными по величине главному вектору. Тогда плечо пары сил определим по формуле

    Повернем пару сил, чтобы ее силы были параллельны главному вектору , а точку приложения силы пары, противоположной по направлению главному вектору, совместим с центром приведения . Тогда

    Так как , то такую систему сил можно отбросить.

    Итак, систему сил, приведенную к силе с парой сил, в том случае, когда и , можно упростить и привести к одной силе —равнодействующей заданной системы сил, отстоящей от центра приведения на расстоянии

    Равнодействующую силу , приложенную к твердому телу, можно перенести в любую точку линии ее действия. Случай, когда , возможен, если за центр приведения взять точку, лежащую на линии действия равнодействующей силы .

    Случай приведения к паре сил

    Если при приведении плоской системы су л к какому-либо центру окажется, что главный вектор , а главный момент , то такую плоскую систему сил можно привести к одной паре сил, алгебраический момент которой равен главному моменту системы сил относительно центра приведения, и в этом случае главный момент не зависит от выбора центра приведения.

    Если главный вектор равен нулю при приведении к одному какому-либо центру, то он равен нулю и при приведении к любому другому центру, так как главный вектор, являясь векторной суммой сил системы, не зависит от выбора центра приведения. Главный момент не зависит от центра приведения только в том случае, когда . В других случаях главный момент системы зависит от выбора центра приведения. Если бы при главный момент зависел от центра приведения, то одна и та же плоская система сил была бы эквивалентна парам сил, имеющим разные алгебраические моменты, что невозможно, так как эквивалентные пары сил, лежащие в одной плоскости, имеют одинаковые алгебраические моменты.

    Таким образом, рассмотрены случаи, которые возможны при приведении плоской системы сил к какому-либо центру. Если и , то система сил находится в равновесии; если , a , или , , то система сил приводится к одной равнодействующей силе; если , , то система приводится к одной паре сил.

    Теорема о моменте равнодействующей силы (Теорема Вариньона)

    Для случая, когда любая система сил, приложенных к твердому телу, плоская или пространственная, приводится к равнодействующей силе, часто применяют так называемую теорему Вариньона: векторный момент равнодействующей рассматриваемой системы сил относительно любой точки равен сумме векторных моментов всех сил этой системы относительно той же точки.

    Рис. 41

    Пусть на твердое тело действует любая система сил (рис. 41), имеющая равнодействующую , т. е.

    Добавим к заданной системе сил ее уравновешивающую силу , которая равна по модулю, но противоположна по направлению равнодействующей силе и имеет с ней общую линию действия. Тогда

    т.е. при добавлении к системе сил уравновешивающей силы, согласно определению уравновешивающей силы, образуется новая система сил, эквивалентная нулю и, следовательно, удовлетворяющая условиям равновесия системы сил, приложенных к твердому телу. В частности, сумма векторных моментов сил этой новой системы сил относительно любой точки равна нулю:

    так как и — две равные и противоположно направленные силы, действующие вдоль одной прямой. Подставляя (5) в (4), получаем

    откуда следует теорема Вариньона

    Если правую и левую части векторного равенства (6) спроецировать на произвольную ось , проходящую через точку , то, учитывая связь момента силы относительно оси с проекцией векторного момента относительно точки на оси, получим теорему Вариньона относительно оси :

    т. е. момент равнодействующей силы относительно произвольной оси равен сумме моментов сил системы относительно той же оси.

    Для случая плоской системы сил, если точку выбрать в плоскости действия сил, из (6) получаем

    Это теорема Вариньона для плоской системы сил: алгебраический момент равнодействующей плоской системы сил относительно любой точки, лежащей в плоскости действия сил, равен сумме алгебраических моментов всех сил этой системы относительно той же точки.

    Различные формы условий равновесия плоской системы сил

    Получены общие условия равновесия плоской системы сил, действующих на твердое тело, в следующей форме:

    Условия равновесия (9) назовем условиями равновесия плоской системы сил в первой форме.

    Условия равновесия плоской системы сил, приложенных к твердому телу, можно сформулировать в других эквивалентных формах. Существуют еще две эквивалентные формы необходимых и достаточных условий равновесия.

    Рассмотрим эти условия равновесия в виде теоремы о трех моментах и третьей формы условий равновесия.

    Теорема о трех моментах (вторая форма условий равновесия)

    Для равновесия плоской системы сил, приложенных к твердому телу, необходимо и достаточно, чтобы суммы алгебраических моментов сил системы относительно трех любых точек, расположенных в плоскости действия сил и не лежащих на одной прямой, были равны нулю, т. е.

    Необходимость этих условий равновесия плоской системы сил обусловлена тем, что если плоская система сил находится в равновесии, то силы этой системы удовлетворяют условиям равновесия в первой основной форме (9). А тогда из последнего условия (9) следует, что сумма алгебраических моментов сил относительно любой точки (следовательно, и точек , , ) равна нулю (рис. 42).

    Для доказательства достаточности условий (10) для равновесия плоской системы сил, действующих на твердое тело, можно привести следующие рассуждения. Так как главные моменты относительно трех точек , и равны нулю, то для любой из этих точек, взятых за центр приведения, система приводится или к равнодействующей, если главный вектор системы отличен от нуля, или система сил оказывается в равновесии, если главный вектор системы равен нулю. Предположим, что она приводится к равнодействующей силе . Тогда если выбрать за центр приведения точку , то, используя теорему Вариньона (8), согласно (10), получим

    Рис. 42

    Выбрав за центр приведения точку , аналогично имеем

    Эти условия для равнодействующей силы , отличной от нуля, могут выполняться в том случае, если линия действия равнодействующей силы проходит через точки и .

    Из последнего условия (10) после применения теоремы Вариньона получаем

    Но , так как точка не находится на прямой, проходящей через точки и . Следовательно, равнодействующая сила равна нулю, что и является достаточным условием равновесия плоской системы сил, приложенных к твердому телу.

    Третья форма условий равновесия

    Условия равновесия плоской системы сил можно сформулировать и так: для равновесия плоской системы сил, приложенных к твердому телу, необходимо и достаточно, чтобы суммы алгебраических моментов сил относительно двух любых точек, лежащих в плоскости действия сил, были равны нулю и алгебраическая сумма проекций этих сил на какую-либо ось плоскости, не перпендикулярную прямой, проходящей через две моментные точки, также была равна нулю, т. е.

    где за ось принята любая прямая, не перпендикулярная . Необходимость условий (11) для равновесия плоской системы сил следует из первой формы условий равновесия (9). Первая часть теоремы о достаточности условий (11) для равновесия (линия действия равнодействующей силы проходит через точки и ) доказывается так же, как и в теореме о трех моментах.

    Из последнего условия (11) (рис.43) следует, что

    так как ось не перпендикулярна прямой, проходящей через точки и . Следовательно, равнодействующая сила равна нулю, что и доказывает достаточность условий (11) для равновесия плоской системы сил, приложенных к твердому телу.

    В частном случае плоской системы параллельных сил можно сформулировать другую форму условий равновесия этой системы сил: для равновесия плоской системы параллельных сил, приложенных к твердому телу, необходимо и достаточно, чтобы суммы алгебраических моментов сил относительно двух любых точек, лежащих в плоскости сил, были равны нулю, т. е.

    Точки и нельзя брать на прямой линии, параллельной силам.

    При применении условий равновесия (12) удобно за момент-ные точки и брать точки, через которые проходят искомые силы, например реакции связей. В этом случае получаются такие уравнения для определения искомых сил, в каждое из которых входит только по одной неизвестной силе; эти уравнения, как правило, решаются проще, чем уравнения, в каждое из которых входят обе неизвестные силы.

    Рис. 43

    Статически определимые и статически неопределимые задачи

    Для любой плоской системы сил, действующих на твердое тело, имеется только три независимых условия равновесия, каждое из которых не является следствием двух других. Независимые условия равновесия можно брать в трех различных формах.

    Следовательно, для любой плоской системы сил из условий равновесия можно найти не более трех неизвестных, а для плоских систем параллельных и сходящихся сил — не более двух неизвестных. Если в какой-либо задаче число неизвестных окажется больше числа независимых условий равновесия, то такую задачу нельзя решить методами статики без рассмотрения прежде всего деформаций тела, т. е. без отказа от основной гипотезы статики об абсолютно твердом теле.

    Задачи, в которых число неизвестных не больше числа независимых условий равновесия для данной системы сил, приложенных к твердому телу, называют статически определимыми. Для любой плоской системы сил, приложенных к твердому телу, в статически определимой задаче число неизвестных должно быть не больше трех, а для плоских систем параллельных и сходящихся сил — не больше двух.

    Пример простейшей статически неопределимой задачи приведен на рис. 44, где представлена балка заданной длины, закрепленная на концах с помощью двух неподвижных цилиндрических шарниров и . На балку действуют активные силы и . Известны также и точки приложения этих сил. Так как для цилиндрического шарнира имеются две неизвестные, например составляющие силы реакции по осям координат, то число неизвестных будет четыре, а независимых условий равновесия можно составить только три.

    Чтобы сделать задачу статически определимой, надо балку на одном конце закрепить, например с помощью так называемой катко-вой опоры. Тогда одна неизвестная будет равна нулю; если катковая опора находится в точке и плоскость опоры катков параллельна оси , то сила равна нулю.

    Рис. 44

    Равновесие системы тел

    Рассмотрим равновесие сил, приложенных к системе нескольких взаимодействующих между собой тел. Тела могут быть соединены между собой с помощью шарниров, соприкасаться друг с другом и взаимодействовать одно с другим, вызывая силы взаимодействия. Такую систему взаимодействующих тел иногда называют сочлененной системой тел.

    Силы, действующие на рассматриваемую систему тел, можно разделить на внешние и внутренние.

    Внешними называют силы, с которыми на тела рассматриваемой системы действуют тела, не входящие в эту систему.

    Внутренними называют силы взаимодействия между телами рассматриваемой системы.

    Если, например, рассматриваемой системой тел является железнодорожный поезд, то внешними силами являются силы веса вагонов и тепловоза, действие рельсов на колеса вагонов и тепловоза, силы сопротивления воздуха. Внутренними силами являются натяжения в стяжках, сила давления газа и т. п.

    Силы веса для любой системы тел, в которую не входит Земля, всегда являются внешними.

    При рассмотрении равновесия сил, приложенных к системе тел, можно мысленно расчленить систему тел на отдельные твердые тела и к силам, действующим на эти тела, применить условия равновесия, полученные для одного тела. В эти условия равновесия войдут как внешние, так и внутренние силы системы тел. Внутренние силы на основании аксиомы о равенстве сил действия и противодействия в каждой точке сочленения двух тел образуют равновесную систему сил (силы и , рис. 45). Поэтому внешние силы, действующие на систему тел отдельно, без внутренних сил, удовлетворяют условиям равновесия сил, приложенных к твердому телу, за которое следует принять эту систему тел.

    Рис. 45

    Покажем это на примере системы двух тел и плоской системы сил (рис. 45). Если составить условия равновесия для каждого твердого тела системы тел, то для тела

    для тела

    Кроме того, из аксиомы о равенстве сил действия и противодействия для двух взаимодействующих тел имеем

    Если сложить (13) и (14), учитывая (15 и (16), то

    Представленные равенства и есть условия равновесия внешних сил, действующих на систему двух тел.

    Для системы тел в том случае, когда на каждое тело действует любая плоская система сил, можно составить условий равновесия и, следовательно, определить неизвестных. Если число неизвестных больше , то задача является статически неопределимой. В случае статически определимой задачи условий равновесия можно получить, если составлять их для каждого тела отдельно, учитывая и силы взаимодействия тел, или составлять условия равновесия для любых комбинаций групп тел, в том числе и для всей рассматриваемой системы тел. При этом внутренние силы для отдельных групп тел учитывать не надо.

    Распределенные силы

    В статике рассматривают силы, приложенные к твердому телу в какой-либо его точке, и поэтому такие силы называют сосредоточенными. В действительности обычно силы бывают приложены к какой-либо части объема тела или его поверхности, а иногда к некоторой части линии. Так как все аксиомы и теоремы статики формулируются для сосредоточенных сил, приложенных к твердому телу, то необходимо рассмотреть способы перехода от распределенных сил к сосредоточенным в простейших, наиболее часто возникающих случаях.

    Распределенные силы прежде всего характеризуются интенсивностью распределенной силы, т.е. силой, приходящейся на единицу объема, поверхности или длины линии. В основном встречаются параллельные и сходящиеся распределенные силы. К параллельным силам, распределенным по объему тела, относится вес частиц этого тела. Сила давления воды на плотину относится к распределенным параллельным силам по поверхности плотины. Сила тяжести частиц тонкой проволоки характеризует распределенные силы по длине линии.

    Рассмотрим замену сосредоточенными силами только распределенных сил по длине линии, т. е. линейных распределенных сил. Для простоты возьмем случаи, когда отрезок линии, по которому распределены силы, является отрезком прямой, а интенсивность этих сил или постоянна (силы распределены по прямоугольнику), или распределена по линейному закону, в простейшем случае — по треугольнику. Комбинируя эти два случая, можно получить линейное распределение интенсивности распределенной силы в более общем случае.

    Параллельные силы постоянной интенсивности, распределенные по отрезку прямой линии

    Пусть на участке прямой линии длиной распределены параллельные силы, интенсивность которых постоянна (рис. 46, а). Заменим эти распределенные силы сосредоточенными. Для этого отрезок разобьем на отрезки достаточно малых размеров по сравнению с его длиной. На каждый такой малый отрезок действует сила которую при достаточной малости длины отрезка можно считать сосредоточенной силой. Заменяя полученную таким образом систему сосредоточенных параллельных сил одной равнодействующей силой, получим

    Рис. 46

    Равнодействующая параллельна распределенным силам и приложена вследствие симметрии распределения сил в середине отрезка .

    Если параллельные силы постоянной интенсивности распределены по отрезку прямой, наклоненному к распределенным силам, то модуль равнодействующей таких сил равен . Линия действия ее, параллельная распределенным силам, проходит через середину отрезка (рис. 46, б). Модуль равнодействующей в этом случае не равен площади параллелограмма, образованного прямой и распределенными силами.

    Параллельные силы, распределенные по отрезку прямой с интенсивностью, изменяющейся по линейному закону

    Рассмотрим распределенные параллельные силы, изменяющиеся по линейному закону (рис. 47, а). Обычно считают, что такие силы распределены по треугольнику. Параллельные распределенные по треугольнику силы приводятся к равнодействующей , по модулю равной

    где — наибольшая интенсивность силы. Это легко можно проверить путем сложения параллельных сосредоточенных сил , приложенных к каждому элементарному отрезку длиной . Наиболее просто это можно сделать путем интегрирования. Действительно,

    Рис. 47

    Если отсчитывать от точки , то из подобия треугольников имеем

    После этого, вставляя под интеграл вместо его значение, получаем

    Точка приложения равнодействующей силы смещается в сторону, где интенсивность силы больше, и совпадает с центром тяжести площади треугольника, который находится в точке пересечения медиан, расположенной на расстоянии от основания треугольника и от его вершины , т. е. . Точку приложения равнодействующей силы можно также определить вычислив момент элементарных сосредоточенных сил , например относительно точки , и применив затем теорему Вариньона о моменте равнодействующей силы.

    Заменяя его значением , получаем

    Учитывая, что найдем

    Если параллельные силы с интенсивностью, изменяющейся по линейному закону, распределены по отрезку прямой, наклоненному к направлению сил (рис. 47, б), то их равнодействующая и делит отрезок так же, как и в том случае, когда распределенные силы перпендикулярны отрезку . Величина равнодействующей в этом случае не равна площади треугольника, образованного отрезком прямой и распределенными силами.

    В более сложных случаях распределенных сил равнодействующую силу и ее точку приложения обычно определяют путем интегрирования и применения теоремы Вариньона. Величину равнодействующей в случае непараллельных распределенных сил находят так же, как и для параллельных, только суммируют (и, следовательно, интегрируют) не элементарные сосредоточенные силы , а их проекции на оси координат. По проекциям уже вычисляют равнодействующую силу и косинусы ее углов с осями координат.

    Реакция заделки

    Пусть имеем тело, например балку , один конец которой заделан в стену (рис. 48, а). Такое крепление конца балки называют заделкой в точке . Пусть на балку действует плоская система сил . Определим силы, которые надо приложить в точке (сечении) балки, если часть балки отбросить.

    К части балки при освобождении ее от заделки в стене приложены распределенные силы. Если эти силы заменить элементарными сосредоточенными силами и затем привести их к точке , то в точке получим силу (главный вектор элементарных сосредоточенных сил ) и пару сил с моментом (главный момент относительно точки элементарных сил ) Момент называют моментом заделки.

    Таким образом, заделка в отличие от шарнира создает не только не известную по величине и направлению реакцию , но еще и пару сил с не известным заранее моментом в заделке (рис. 48, б).

    Очевидно, если рассмотреть любую часть балки, расчленив ее мысленно по сечению , то в месте расчленения надо приложить неизвестные силу и пару сил, заменяющие действие отброшенной части балки на рассматриваемую ее часть, причем сила и момент пары сил, действующие на различные части балки, будут иметь противоположные направления действия и вращения соответственно, как всякое действие и противодействие.

    Рис. 48

    Решение задач на равновесие плоской системы сил, приложенных к твердому телу и системе тел

    Рассмотрим общие положения о решении задач на равновесие плоской системы сил, действующих на одно твердое тело и на систему тел. Весь процесс решения задачи на равновесие сил можно расчленить на ряд этапов, которые характерны для большинства задач.

    К выбранному для рассмотрения телу или системе тел надо приложить все действующие силы, как активные, так и реакции связей; если нужно, расчленить систему тел на отдельные тела или группы тел. Если связью является абсолютно гладкая поверхность какого-либо тела, то реакция связи в этом случае направлена по нормали к общей касательной в точке соприкосновения в сторону, противоположную тому направлению, в котором связь препятствует перемещению рассматриваемого тела.

    Если связью является цилиндрический шарнир, позволяющий телу вращаться вокруг его оси, то реакцию шарнира, лежащую в плоскости, перпендикулярной оси, следует разложить на две заранее не известные составляющие по положительным направлениям осей координат. Если эти составляющие после их определения из уравнений равновесия будут иметь знак минус, то составляющие реакции направлены противоположно положительному направлению осей координат.

    Все гибкие связи (канаты, тросы, ремни и т. п.) создают реакции, направленные по касательной к гибкой связи в данной точке.

    Если связью является заделка, которая в отличие от цилиндрического шарнира не позволяет телу поворачиваться, то кроме двух неизвестных составляющих реакций в этой точке надо еще приложить пару сил с не известным заранее моментом заделки.

    Эти же случаи связей возможны и при расчленении систем тел.

    Выявление всех сил, действующих на рассматриваемое тело или систему тел, особенно правильная замена различных видов связей их реакциями, является одним из главных этапов при решении задач на равновесие.

    При расчленении системы тел надо следить, чтобы силы взаимодействия между телами или группами тел сочленной системы в точках сочленения были равны по модулю, но противоположны по направлению. При рассмотрении системы тел (или их группы) силы взаимодействия между телами системы (или их группы) прикладывать не нужно, так как эти силы являются внутренними и в уравнения равновесия для системы тел (или группы) не войдут.

    Рис. 49

    После выявления всех сил надо выбрать оси координат и моментные точки, а затем, составив условия равновесия сил в одной из форм, решить полученные уравнения относительно неизвестных.

    Решение уравнений будет более простым, если при их составлении в каждое из уравнений добавляется по одной новой неизвестной. Этого удается достичь, если за моментную точку брать такую, в которой пересекаются две искомые силы. Такой точкой обычно является цилиндрический шарнир. Оси координат надо брать так, чтобы одна или две неизвестные силы были перпендикулярны одной из осей координат и, следовательно, параллельны другой оси. В этом случае в соответствующее условие равновесия для одного тела войдет только одна неизвестная сила.

    Приведем примеры решения задачи на плоскую систему сил.

    Пример 1.

    Дана система двух твердых тел, соединенных с помощью шарнира (рис.49). Балка , изогнутая под прямым углом, имеет заделку в точке . Круговая арка закреплена в точке с помощью стержня, имеющего на концах шарниры. Размеры тел и приложенные силы указаны на рисунке. Дуговая стрелка условно обозначает пару сил. Силами тяжести тел пренебречь. Определить силы реакций в точках и .

    Решение. Заменим распределенные силы сосредоточенными. Величина равнодействующей силы (рис. 50) распределенных по треугольнику сил на участке определяется по формуле

    Точка приложения силы отстоит от точки на , т.е. на 1 м. Значение равнодействующей распределенных по арке радиальных сил определяем как произведение длины хорды , стягивающей дугу , на интенсивность распределенных сил , т. е.

    Рис. 50

    Линия действия равнодействующей силы вследствие симметрии распределения сил проходит через центр арки , деля угол, стягивающий арку, на равные части.

    Рассмотрим сначала равновесие системы двух тел, состоящих из балки и арки . На эту группу тел действуют силы пара сил с моментом , силы реакций в заделке и в опоре .

    Реакции заделки в точке в общем случае дают три неизвестные: две составляющие силы по осям координат и момент пары сил; одна неизвестная сила имеется в точке . Ее дает шарнирный стержень. Таким образом, имеем четыре неизвестные, а независимых уравнений для их определения — только три. Систему тел следует расчленить на отдельные тела (рис. 51), приложив к каждому из них в точке силы действия одного тела на другое, которые равны по величине, но противоположны по направлению.

    В дальнейшем целесообразно на рисунках у стрелок, изображающих силы, ставить только буквы, обозначающие значения сил, без знака вектора над ними (рис. 51). Это уменьшит число неизвестных и, следовательно, количество уравнений для их определения.

    Всего имеется шесть неизвестных, считая составляющие силы реакции в шарнире . Составляя по три уравнения равновесия сил для каждого тела, можно получить шесть уравнений для нахождения из них всех неизвестных. Требуется определить только четыре неизвестные реакции в точках и . Поэтому составим уравнения так, чтобы в них не входили реакции в точке и по возможности в каждое уравнение входило не более одной новой неизвестной.

    Рис. 51

    Составим для арки одно условие равновесия сил в форме суммы моментов сил относительно точки . Имеем

    откуда получаем .

    После этого для всей системы тел применим условие равновесия в форме суммы проекций сил на оси и . Получим

    откуда .

    Для определения момента пары сил в заделке достаточно применить для тела условие равновесия в форме суммы моментов сил относительно точки . Имеем

    откуда .

    Если дополнительно требуется определить силы и , то следует применить условия равновесия для тела в форме проекций сил на оси и . Тогда

    Из этих уравнений получаем

    Для контроля правильности определения реакций в точках и следует составить условие равновесия, например, в форме суммы моментов сил относительно точки для всей системы. Полученные ранее значения неизвестных должны обратить его в тождество.

    Задача считается решенной, если известны проекции искомых сил на оси координат, так как по проекциям легко определяются модули этих сил и косинусы углов сил с осями координат.

    Пример 2.

    Для системы тел, находящихся в равновесии, определить реакцию шарнира (рис. 52). Необходимые данные указаны на рисунке. Стержни и , блоки и нить считать невесомыми. Трением в шарнирах пренебречь. Дуговой стрелкой обозначена пара сил, — модуль алгебраического момента.

    Рис. 52

    Решение. Рассмотрим всю систему тел, освободив ее от связей, т.е. от цилиндрических шарниров в и . Неизвестные по величине и направлению силы реакций этих шарниров разложим на составляющие предположив, что они направлены по положительному направлению осей координат. Неизвестных четыре, а условий равновесия сил для всей системы тел можно составить только три. Поэтому рассмотрим другие комбинации тел или отдельные тела.

    Для определения удобно составить условие равновесия для всей системы тел в форме суммы моментов сил относительно точки . Имеем

    откуда . Из приведенного уравнения получилось со знаком плюс; следовательно, предположение о первоначальном направлении в положительную сторону оси оказалось правильным.

    Рис. 53

    Другие условия равновесия сил для всей системы тел не позволяют определить неизвестную , так как в уравнения войдет неизвестная сила .

    Рассмотрим отдельно равновесие стержня (рис. 53), освободив его от связей. В шарнире неизвестную силу реакции заменим составляющими, направленными параллельно осям координат в положительную сторону. В точке приложим силу натяжения отброшенной нити, которая по величине равна силе тяжести груза и направлена по нити.

    Для определения составим условие равновесия для сил, приложенных к стрежню , в форме суммы моментов сил относительно точки . В это условие не войдут неизвестные силы и , которые определять не требуется. Имеем

    Отсюда находим . Знак плюс у этой силы указывает на правильность предположения о направленности .

    Для приобретения опыта силового анализа в системах тел рассмотрим дополнительно еще несколько вариантов частей системы тел и отдельных тел с приложенными к ним силами (рис. 54. 57).

    Рис. 54

    Рис. 55

    Рис. 56

    Рис. 57

    При замене отбрасываемых тел силами учтено, что оси блоков и являются цилиндрическими шарнирами и реакции от них следует разлагать на составляющие, параллельные осям координат. Рассматривая силы, с которыми тела действуют друг на друга, следует учитывать, что, согласно аксиоме статики, силы действия и противодействия равны по величине, но противоположны по направлению. Так, если стержень действует на блок в точке с силами и , направленными в положительные стороны осей координат (рис. 56), то блок будет действовать на стержень (рис. 57) с силами, равными по модулю, но направленными в противоположные стороны.

    При отбрасывании нити следует учитывать, что ее натяжение во всех точках при отсутствии трения в осях блоков одинаково по величине и направлено по касательной к нити. Нить при этом должна испытывать только растяжение. При рассмотрении отдельного блока силы натяжения нитей следует приложить в двух точках, в которых отбрасываются части нити.

    Теорема Вариньона

    Из формулы, определяющей расстояние от центра приведения до линии действия равнодействующей,

    (см. рис. 74) можно вывести уравнение, выражающее теорему Вариньона для произвольной плоской системы сил:

    момент равнодействующей относительно любой точки равен алгебраической сумме моментов заданных сил относительно той же точки.

    Теорема Вариньона находит широкое применение при решении задач по статике, в частности во всех тех задачах, где рассматривается равновесие рычага.

    При помощи теоремы Вариньона очень просто определяется равнодействующая какого угодно числа параллельных сил (рис. 80).

    Известно, что модуль равнодействующей любой плоской системы сил равен модулю главного вектора:

    Но если в данном случае расположить оси проекции так, как показано на рис. 80, одну ось — перпендикулярно к силам, а другую—параллельно им, то

    Таким образом, модуль равнодействующей, параллельной системы сил равен абсолютному значению алгебраической суммы проекций сил на ось, параллельную этим силам.

    Так как =0, то вектор равнодействующей направлен параллельно составляющим силам. Сторона, в какую направлен R, определяется по знаку Если у алгебраической суммы проекций получается знак «плюс», то равнодействующая направлена в сторону положительного направления оси; если получается знак «минус», то равнодействующая направлена противоположно положительному направлению оси.

    Определив модуль и направление равнодействующей, по теореме Вариньона находим расстояние ОА, на котором расположена

    KL- линия действия R от произвольно выбранного центра моментов О.

    Задача 1.

    Определить равнодействующую двух параллельных сил направленных в одну сторону (рис. 81, о), если

    1. Примем за начало осей проекций точку А. Ось х расположим перпендикулярно к данным силам и направим ее вправо, а ось у направим вдоль силы вниз (рис. 81,6).

    2. Найдем модуль равнодействующей:

    Так как сумма проекций положительна, то вектор равнодействующей направлен тоже вниз.

    3. Приняв за центр моментов точку А, найдем расстояние АС от точки A до линии действия равнодействующей.

    В данном случае



    Таким образом, равнодействующая двух данных сил численно равна 27 н, и линия ее действия расположена от точки А на расстоянии АС = 1 м (рис. 81, в).

    Задача 2.

    Найти равнодействующую двух параллельных сил направленных в разные стороны, если = 12 кн и = 60 кн (рис. 82, а).

    1. Расположим оси Ох и Оу так, как показано на рис. 82, б.

    2. Найдем модуль равнодействующей:

    .
    Сумма проекций заданных сил имеет отрицательное значение. Следовательно, равнодействующая направлена влево (ось Ох направлена вправо).

    3. Приняв за центр моментов точку О и предположив, что линия действия R пересекает отрезок ОВ в точке А, составим уравнение

    .


    Числовое значение О А получается отрицательным, значит этот отрезок от точки О необходимо отложить в противоположную сторону от ранее предполагаемого.

    Равнодействующая заданных сил численно равна 48 и, направлена влево, и линия ее действия лежит ниже точки О на 0,25 м (рис. 82, в).

    Задача 3.

    К концам прямолинейной однородной планки длиной 1,6 м и весом 5 н прикреплены два груза (рис. 83): слева —груз = 20 н, справа — = 15 н. В каком месте планки нужно приделать петельку, чтобы подвешенная на ней планка с грузами оставалась в горизонтальном положении?

    1. Изобразим на рис. 83 в горизонтальном положении планку АВ с грузами Так как планка однородная, ее вес G —5 н приложен в середине (в точке С).

    Таким образом, к планке приложена система трех параллельных сил, действующих в одну сторону (рис. 83, б).

    2. Оси проекций расположим, как показано на рис. 83, б.

    3. Найдем модуль равнодействующей сил

    Равнодействующая направлена вертикально вниз.

    4. Определим, на каком расстоянии AD от точки А (левого конца планки) расположена линия действия равнодействующей:

    Линия равнодействующей проходит через точку D на расстоянии 0,7 м от левого конца планки.

    В этом месте и необходимо прикрепить к планке петельку. Если теперь за петельку подвесить планку на гвоздь или прикрепить к нити, то планка будет находиться в равновесии, оставаясь горизонтальной, так как равнодействующая R уравновесится реакцией гвоздя или нити.

    Задача 4.

    Балансир АВ, на который действуют пять горизонтально направленных параллельных сил (рис. 84), должен находиться в равновесии в вертикальном положении, будучи насаженным на горизонтальную ось.

    Определить, где необходимо поместить ось балансира, пренебрегая его весом.

    1. Расположив оси проекций, как указано на рис. 84, найдем модуль равнодействующей системы параллельных сил:

    Таким образом, равнодействующая направлена вправо.

    2. Определим расстояние ВО от нижнего конца балансира до линии действия из уравнения Вариньона (центр моментов в точке В):


    Следовательно, линия действия равнодействующей пересекает находящийся в вертикальном положении балансир на расстоянии 64,5 см от нижнего конца В. Здесь (в точке О) и нужно поместить ось балансира.

    Следующую задачу рекомендуется решить самостоятельно.

    Задача 5.

    Где необходимо поместить ось балансира, описанного в предыдущей задаче, если силу =15 кн направить в противоположную сторону?

    Ответ. ВО = 29,5 см.

    Задачи, приведенные ниже, решаются при помощи так называемого условия равновесия рычага, непосредственно вытекающего из теоремы Вариньона.

    Рычагом можно назвать любое тело, поворачивающееся либо вокруг закрепленной оси, либо около линии контакта, образующейся при свободном направлении на другое тело.

    Находясь под действием сил, рычаг уравновешен лишь в том случае, если линия действия равнодействующей пересекает ось или линию опоры. Причем если опорой рычага АВ служит закрепленная ось (неподвижный шарнир), то линия действия равнодействующей может быть направлена к рычагу под любым углом а (рис. 85, а). Если же рычаг АВ свободно опирается на идеально гладкую опору

    (рис. 85, б), то линия действия равнодействующей должна быть перпендикулярна к опорной поверхности.

    В любом нз этих случаев равновесие возникает потому, что система сил, действующих на рычаг, уравновешивается реакцией опоры численно равной равнодействующей. А так как момент равнодействующей относительно опоры равен нулю, то из выражения теоремы Вариньона следует уравнение

    выражающее условие равновесия рычага.

    Задача 6.

    Масса неоднородного стержня составляет 4,5 кг. Для определения положения центра тяжести стержня его левый конец положен на гладкую опору, а правый зацеплен крюком динамометра (рис. 86, а). При горизонтальном положении стержня динамометр показывает усилие 1,8 кГ. Расстояние АВ —130 см от левой опоры до динамометра определено путем непосредственного измерения. Определить ^положение центра тяжести стержня.

    1. Рассмотрим стержень как рычаг с опорой в точке А. Кроме реакции опоры, на него действуют две нагрузки: вес G = 4,5 кГ (1 кг массы притягивается к земле силой, равной 1 кГ), приложенный в центре тяжести на искомом расстоянии х от опоры А, и усилие пружины динамометра Я = 1,8 кГ (рис. 86, б).

    2. Составим уравнение равновесия рычага:

    В данном случае относительно точки А моменты создают две силы и G:


    Решаем полученное уравнение:

    Центр тяжести стержня расположен на расстоянии 52 см от левой опоры.

    Задача 7.

    Какова должна быть масса однородной доски (рис. 87, а), чтобы, опираясь в точке В на гладкую опору, она с положенными на нее грузами =100 кг и = 48 кг находилась в равновесии? Центр тяжести доски расположен в точке С.

    1. Рассматривая доску как рычаг, видим, что на нее действуют гри нагрузки: вес левого груза вес правого груза

    и собственный вес доски (рис. 87, б).

    2. Для равновесия доски необходимо, чтобы алгебраическая сумма моментов этих сил относительно опоры В равнялась нулю. Следовательно,

    3. Подставив вместо весов их выражения через массы и разделив обе части равенства на постоянную величину g (ускорение свободного падения 9,81 получим

    4. Отсюда находим массу доски:

    Масса доски 8 кг.

    Задача 8.

    Предохранительная заслонка открывается в тот момент, когда давление в резервуаре превышает внешнее атмосферное на р=150 Заслонка прижимается к отверстию в резервуаре коленчатым рычагом АВС (рис. 88).

    На каком расстоянии х от опоры рычага необходимо поместить груз весом G = 120 н, чтобы заслонка открылась при заданном давлении, если площадь отверстия в резервуаре а =12 см. Весом рычага пренебречь.

    1. На рычаг АВС предохранительного устройства действуют две нагрузки: вес груза G = 120 н и сила Р, открывающая заслонку:

    2. Условие равновесия рычага выразится уравнением


    3. Решая это уравнение, находим

    Груз необходимо поместить на расстоянии 30 см от опоры В.

    Задача 9.

    На рис. 89, а изображен коленчатый рычаг АВС, к короткому колену которого при помощи нити прикреплен груз массой = 50 кг, а к длинному — груз массой = 10 кг.

    Под каким углом а к длинному колену необходимо расположить вторую нить, чтобы нить, удерживающая первый груз, образовала с АВ угол 30°? Расстояния

    Считать, что при этом положении рычага линия действия собственного веса рычага проходит через ось В опорного шарнира рычага.

    1. На рис. 89, б изобразим расчетную схему рычага; к точке А отвесно приложен вес первого груза к точке С под искомым углом а к СВ приложен вес второго груза Вес рычага приложен в точке В.

    2. Замечая, что (так как плечо силы равно нулю), составим уравнение равновесия рычага:

    3. Выразив плечи BD и BE через длины колен рычага, а веса и — через массы, получим уравнение

    Этому значению sin а соответствует прямой угол. Следовательно,

    Поэтому нить, удерживающую второй груз, нужно расположить перпендикулярно к длинному колену рычага.

    Следующую задачу рекомендуется решить самостоятельно.

    . Однородный стержень АВ длиной 2 м и весом 100 н прикреплен шарниром А к вертикальной стене АЕ (рис. 90). Под каким углом а к стержню должна быть направлена веревка с грузом Р = 50 н на конце, перекинутая через блок D, чтобы стержень находился в равновесии, образуя со стеной угол Трением на блоке пренебречь. Ответ, а —60 или 120°.

    Равновесие произвольной плоской системы сил

    Задача на равновесие произвольной плоской системы сил решается по той же общей схеме, которая приведена в § 8-2. Придерживаясь этой схемы, необходимо учитывать следующее.

    Как известно, любую плоскую систему сил можно привести к главному вектору и главному моменту (Е. М. Никитин, § 26).

    Если же система сил уравновешена (тело, находящееся под действием такой системы сил, либо неподвижно, либо равномерно вращается около неподвижной оси, либо находится в равномерном и прямолинейном поступательном движении), то(Е. М. Никитин, § 30). Эти равенства выражают два необходимых и достаточных условия равновесия любой системы сил.

    Для произвольной плоской системы сил из этих двух условий непосредственно получаем три уравнения равновесия:

    Первое и второе выражения — уравнения проекций — образуются из условия третье выражение — уравнение моментов — из условия

    Если на тело действует система параллельных сил, то уравнений равновесия получится только два: уравнение проекций на ось, параллельную силам, и уравнение моментов

    При решении некоторых задач одно или оба уравнения проекций целесообразно заменить уравнениями моментов относительно каких-либо точек, т. е. систему уравнений равновесия можно представить в таком виде:

    или

    В первом случае линия, проходящая через точки А и В, не перпендикулярна к оси х. Во втором случае центры моментов А, В и С не лежат на одной прямой линии.

    Для системы параллельных сил соответственно получаем два уравнения моментов:

    В этом случае точки А и В не лежат на прямой, параллельной силам.

    В задачах, решаемых при помощи уравнений равновесия, обычно рассматриваются тела, находящиеся в состоянии покоя, тогда система сил, действующих на это тело, уравновешена.

    Силы, действующие на тело, делятся на две группы. Одна группа сил называется нагрузками (активные силы), вторая группа сил называется реакциями связей (пассивные силы).

    Нагрузки, как правило, бывают заданы. Они имеют числовое значение, точку приложения к телу и направление их действия.

    В рассматриваемых ниже задачах используются лишь три разновидности нагрузок: сосредоточенные силы, равномерно распределенные силы * и пары сил (статические моменты) **.

    Сосредоточенными называются силы, приложенные к точке тела. Если, например, на тело действуют нагрузки как пока-

    заново на рис. 91, а, действия этих нагрузок можно считать приложенными соответственно к точкам А или В тела и на расчетных схемах изобразить так, как это выполнено на рис. 91, б.

    Равномерно распределенные нагрузки, например кирпичная кладка (рис. 92, а), или собственный вес однородного тела (бруса, балки) постоянного поперечного сечения по всей его длине задается при помощи двух параметров —интенсивности q и длины l на протяжении которой они действуют. На расчетных схемах эти нагрузки изображаются так, как показано на рис. 92, б.

    * К распределенным нагрузкам относятся также неравномерно распределенные нагрузки, но в настоящем пособии они не рассматриваются.
    ** Здесь не рассматриваются случаи, когда пары сил действуют на некотором расстоянии непрерывной цепочкой моментов (распределенные моменты).

    Пара сил (сосредоточенный момент), например, может быть образована двумя одинаковыми грузами Р, действующими на тело так, как показано на рис. 93, а. Условное изображение пары сил, действующей на тело, показано на рис. 93, б.

    Очень часто в каком-либо месте тела возникает совместное действие сосредоточенной силы и момента. Пусть, например, груз Q подвешен на конце бруса, жестко заделанного другим концом

    в каком-либо теле (рис. 94, а). Если перенести действие силы в точку А тела (рис. 94, б), то получим в ней совместное действие сосредоточенной силы и момента.

    Как правило, в задачах по статике реакции связей —искомые величины. Для каждой искомой реакции связи обычно необходимо

    знать ее направление и числовое значение (модуль).

    Направления реакций идеальных связей — связей без трения — определяют в зависимости от вида связи по следующим правилам.

    1. При свободном опирании тела на связь реакция связи направлена от связи к телу перпендикулярно либо к поверхности тела либо к поверхности связи рис. 95), либо к общей касательной обеих поверхностей рис. 95).

    Во всех этих случаях связь препятствует движению тела в одном направлении —перпендикулярном к опорной поверхности.

    2. Если связями являются нити, цепи, тросы (гибкая связь), то они препятствуют движению тела только будучи натянутыми.

    Поэтому реакции нитей, цепей, тросов всегда направлены вдоль их самих в сторону от тела к связи (рис. 96).

    3. Если связь тела с какой-либо опорной поверхностью осуществляется при помощи подвижного шарнира (рис. 97), то его реакция направлена перпендикулярно к опорной поверхности. Таким

    образом, подвижный шарнир (т. е. шарнир, ось которого может передвигаться вдоль опорной поверхности) представляет собой конструктивный вариант свободного опирания.

    4. Если соединение тела со связью осуществляется при помощи неподвижного шарнира (рис. 98), то определить непосредственно направление реакции нельзя, за исключением тех частных случаев, которые описаны ниже.

    Шарнирное соединение препятствует поступательному перемещению тела во всех направлениях в плоскости, перпендикулярной к оси шарнира. Направление реакции неподвижного шарнира может быть любым в зависимости от направления действия остальных сил. Потому сначала определяют две взаимно перпендикулярные составляющие реакции шарнира, а затем, если нужно, по правилу параллелограмма или треугольника можно определить как модуль, так и направление полной реакции

    Направление реакции неподвижного шарнира непосредственно определяют в двух следующих случаях:

    • а) если, кроме реакции шарнира, все остальные силы (нагрузки и реакция другой связи) образуют систему параллельных сил, то реакция неподвижного шарнира также параллельна всем силам;
    • б) если, кроме реакции шарнира, на тело действуют еще только две непараллельные силы, то линия действия реакции неподвижного шарнира проходит через ось шарнира и точку пересечения двух других сил (задачи 47-9 и 48-9).

    5. Движение тела может быть ограничено жесткой заделкой в какой-либо опоре (рис. 99). В этом случае даже одна жесткая заделка обеспечивает равновесие тела при любых нагрузках.

    ее реакции заранее определить нельзя и сначала определяют составляющие Кроме того, жесткая заделка препятствует повороту тела в плоскости действия сил, поэтому, кроме силы реакции, на тело действует еще момент заделки уравновешивающий стремление нагрузок повернуть тело (вывернуть тело из заделки).

    Таким образом, если опорой тела является жесткая заделка, то со стороны последней на тело действуют реакция заделки, которую можно заменить двумя взаимно перпендикулярными составляющими, и момент заделки.

    6. Иногда тело удерживается в равновесии при помощи жестких стержней, шарнирно соединенных с телом и с опорами (рис. 100). В отличие от гибкой связи (см. п. 2) такие стержни могут испытывать не только растяжение, но и сжатие.

    Возможны и такие случаи, когда нельзя заранее установить, какие стержни растянуты, а какие сжаты. Поэтому при составлении уравнений равновесия исходят из того, что все стержни растянуты. Если же некоторые стержни окажутся в действительности сжатыми, то в результате решения числовые значения реакций таких стержней получатся отрицательными.

    Задача 10.

    На горизонтальную балку АВ, левый конец которой имеет шарнирно-неподвижную опору, а правый —шарнирноподвижную, в точках С и D поставлены два груза: (рис. 101, а). Определить реакции опор балки.

    1. Рассмотрим равновесие балки АВ, на которую в точках С и D действуют две вертикальные нагрузки (рис. 101, б).

    2. Освободив правый конец балки от связи и заменив ее действие реакцией направленной перпендикулярно к опорной поверхности, увидим, что на балку действует система параллельных сил. Поэтому, если освободить и левый конец балки от шарнирно неподвижной опоры, то се реакция будет также направлена вертикально (рис. 101, б).

    3. Составим систему уравнений равновесия вида (5), приняв для одного уравнения за центр моментов точку А, а для другого — точку В;

    4. Решая уравнения, из (I) находим


    5. Проверим правильность решения, составив уравнение проекций сил на вертикальную ось у:

    Подставляя в это уравнение числовые значения, получаем тождество

    14 — 10 — 20+16=0 или 0 =0

    Значит задача решена правильно.

    При решении задач рекомендуется не пренебрегать проверкой. От правильности определения реакций опор зависит правильность всего остального решения или расчета.

    Задача 11.

    На консольную балку, имеющую в точке А шарнирно-неподвижную, а в точке В шарнирно-подвижную опору, действуют две сосредоточенные нагрузки: 50 кн, как показано на рис. 102, а; угол а=40°. Определить реакции опор балки.

    1. Рассматривая находящуюся в равновесии балку AD, видим, что в точке С на нее действует вертикально вниз нагрузка а в точке D под углом ос к АВ действует другая нагрузка (рис. 102, б).

    2. Освобождаем балку от связен и заменим их действие реакциями. В месте шарнирно-подвижной опоры В возникает вертикальная реакция Направление реакции шарнирно-неподвижной опоры в данном случае непосредственно определить нельзя, поэтому заменим эту реакцию ее двумя составляющими

    3. Для полученной системы из пяти сил, произвольно расположенных в плоскости, составим систему уравнений равновесия вида (3), расположив ось х вдоль балки, а за центры моментов приняв точки А и В:


    4. Решаем полученные уравнения.

    ХА = Р2 cos а = 50 cos 40° = 38,3 кн.

    Так как


    Знак минус, получившийся в последнем случае, показывает, что — вертикальная составляющая реакция неподвижного шарнира— направлена вниз, а не вверх, как предполагалось перед составлением уравнения (3).

    5. При необходимости реакцию шарнира А легко определить (рис. 102, в).

    Модуль реакции шарнира А найдем из формулы

    Направление реакции Ra установим, определив угол

    откуда

    6. Проверим правильность решения задачи. Так как при решении не использовано уравнение проекций на ось у, то используем его для проверки:

    Уравнение составлено по рис. 102, б.

    После подстановки в это уравнение известных значений получим:

    В данном случае, проверка решения при помощи уравнения проекций не дает возможности установить правильность определения полной реакции шарнира А. Чтобы проверить и этот этап решения, составим уравнение моментов относительно точки D, воспользовавшись рис. 102, в, на котором изображена реакция так, как она направлена в действительности:

    Подставляем в это уравнение числовые значения, имея в виду, что

    Расхождение в результатах, равное 0,3, получается из-за округлений при вычислениях.

    В следующих задачах проверка решения не приводится и ее рекомендуется производить самостоятельно.

    Задача 12.

    Горизонтальная балка имеет в точке А шарнирноподвижную опору, плоскость которой наклонена к горизонту под углом а=25° (рис. 103, а), а в точке В — шарнирно-неподвижную опору. Балка нагружена в точках С и D двумя сосредоточенными силами = 24 кн и = 30 н.

    Определить реакции опор.

    1. Так же как и в задаче 75-14, балка нагружена двумя параллельными силами, но в отличие от этой задачи здесь реакция подвижного шарнира направлена не параллельно вертикальным нагрузкам, а под углом а к вертикали — перпендикулярно к опорной поверхности шарнира (рис. 103,6). Поэтому реакция неподвижного шарнира не будет направлена вертикально и, так же как в задаче 76-14, ее целесообразно заменить двумя составляющими

    2. Расположив оси х и у как показано на рис. 103, б, составляем уравнения равновесия вида (1):

    3. Решаем полученные уравнения. Из уравнения (3) находим

    Из уравнения (2) находим

    Из уравнения (1) находим

    Таким образом, реакция шарнира А

    а составляющие реакции шарнира В

    и
    4. Проверку решения производим при помощи уравнения моментов относительно точки С или D.

    Следующую задачу рекомендуется решить самостоятельно.

    Задача 13.

    На консольную балку, имеющую в точке А шарнирно-неподвижную, а в точке В шарнирно-подвижную опору,

    действуют две нагрузки (рис. 104, а): в точке D — сосредоточенная нагрузка Р=8 кн, а на участке СВ — равномерно распределенная нагрузка интенсивностью q — 2 кн/м. Определить реакции опор.

    1. В этой задаче, кроме сосредоточенной силы Р, на участке СВ действует равномерно распределенная сила, интенсивность которой q. Полная величина этой нагрузки (ее равнодействующая) равна q-CB и приложена в точке О посредине участка СВ (рис. 104, б), т. е.


    2. Так же как в задаче 75-14, реакция подвижного шарнира направлена вертикально (перпендикулярно к опорной поверхности). Следовательно, и реакция неподвижного шарнира направлена вертикально. Таким образом, на балку действует система параллельных сил (см. рис. 104, б).

    3. Составим два уравнения моментов относительно точек В и А:

    4. Из уравнения (1)


    Отрицательное значение реакции означает, что она направлена вниз, а не вверх, как показано на рис. 104, б, потому что момент силы Р относительно опоры В больше, чем момент равномерно распределенной нагрузки.

    Из уравнения (2) находим


    Таким образом, реакция шарнира А равна 0,75 кн и направлена вертикально вниз; реакция шарнира В составляет = 14,25 кн и направлена вертикально вверх.

    5. Для проверки решения можно использовать уравнение проекций на вертикальную ось.

    Задача 14.

    На двухконсольную балку с шарнирно-неподвижной опорой в точке Лис шарнирно-подвижной в точке В действуют, как показано на рис. 105,а, сосредоточенная сила Р—10 кн, сосредоточенный момент (пара сил)

    М = 40 кн м и равномерно распределенная нагрузка интенсивностью q — 0,8 кн/м. Определить реакции опор.

    1. В отличие от предыдущей задачи здесь, кроме сосредоточенной силы и равномерно распределенной нагрузки, равнодействующая которой приложена в точке О посредине участка на балку действует
    момент М, направленный по часовой стрелке (рис. 105, б).

    2. После освобождения балки от связей и замены связей их реакциями получаем уравновешенную систему, составленную из четырех параллельных сил и одной пары сил (момента).

    * Перед тем как приступить к рассмотрению этой и следующих задач, необходимо вспомнить два важных свойства нары сил.

    3. Составим два уравнения моментов относительно точек В и А:

    4. Решая эти уравнения, находим, что

    Следующую задачу рекомендуется решить самостоятельно.

    Задача 15.

    Жестко заделанная у левого конца консольная балка АВ (рис. 107, а) нагружена равномерно распределенной

    нагрузкой интенсивностью q 5 сосредоточенной силой P= 12 моментом М = = 20 кн м. Определить реакции заделки.
    Решение.

    1. На балку действуют три нагрузки: в точке С—вертикальная сосредоточенная сила Р, по всей длине балки — равномерно распределенная нагрузка, которую заменим сосредоточенной силой

    приложенной в точке Правый

    конец балки нагружен моментом М, действующим против хода часовой стрелки (рис. 107, б).

    2. Равновесие балки обеспечивается жесткой заделкой у точки А. Освободив балку от связи, заменим ее действие силой — реакцией связи и реактивным моментом Но так как реакцию заделки сразу определить нельзя (по тем же причинам, что и направление реакции неподвижного шарнира), заменим ее составляющими совместив их с осями х и у (см. рис. 107, б).

    3. Составим уравнения равновесия —уравнение проекции на оси х и у и уравнение моментов относительно точки А:

    4. Из уравнения (1)

    а это означает, что горизонтальная составляющая реакции заделки равна нулю, так как в данном случае нет усилий, смещающих балку АВ в горизонтальном направлении.

    Выше найдено, что значит реакция заделки перпендикулярна к оси х. Следовательно,

    5. Проверку правильности решения можно произвести при помощи уравнения моментов относительно точки С или В. В любое из них входят обе найденные величины.

    Следующую задачу рекомендуется решить самостоятельно.

    Задача 16.

    Однородный брус длиной AB = 5 м и весом G = 400 н концом А упирается в гладкий горизонтальный пол и в гладкий вертикальный выступ, а в точке D— в ребро вертикальной стенки высотой ED=4 м. В этом положении брус образует с вертикальной плоскостью стенки угол a = 35° (рис. 109, а). Определить реакции опор.

    1. В отличие от предыдущих задач здесь нет ни шарнирных опор, ни жесткой заделки. Брус свободно опирается о пол, выступ и ребро стенки. Нагрузкой является только вес бруса, приложенный по его середине, так как брус однороден.

    2. Освободив брус от связей, изобразим его вместе со всеми действующими на него силами (рис. 109, б): в точке С на брус действует

    его вес Пренебрегая поперечными размерами бруса, можно считать, что в точке А на брус действуют дв^ реакции: — вертикальная реакция пола и — горизонтальная реакция выступа; в точке D к брусу приложена реакция стенки. В данном случае брус свободно опирается о связи, поэтому реакция связей перпендикулярна к опорным поверхностям.

    3. Таким образом, на брус действуют четыре силы: Расположив оси проекций как показано на рис. 109, б и приняв за центр моментов точку А, составим уравнения равновесия:


    4. Решаем полученную систему уравнений.

    Предварительно определяем АК и AD. Из рис. 109, б находим, что

    И теперь из уравнения (3):

    .

    5. Проверку можно произвести при помощи уравнения моментов относительно точки С.

    Задача 17.

    Однородный брус АВ длиной 5 л и весом G = 180 и, прикрепленный к вертикальной стене шарниром А, опирается в точке D на выступ, ширина которого=1,5 м; при этом брус образует с вертикалью угол а=30°. К концу В бруса прикреплена нить, перекинутая через блок и несущая на другом конце груз Р = 360 н (рис. 110); угол = 40°. Определить реакцию выступа ED и полную реакцию шарнира А.

    1. К брусу АВ приложены две нагрузки—его собственный вес G в середине бруса (так как брус однородный), действующий вертикальную вниз, и к нижнему концу —сила , направленная под углом к В А. Изобразим брус вместе с этими силами отдельно на рис. 111, а.

    2. Брус, имеет две опоры. В точке D он свободно опирается на ребро выступа ED, и поэтому реакция выступа направлена перпендикулярно к брусу АВ. В точке А брус имеет шарнирнонеподвижную опору, направление реакции которой неизвестно. Заменим искомую реакцию двумя составляющими , допустив, что первая направлена горизонтально, а вторая — вертикально (см. рис. 111,о).

    Таким образом, на брус АВ действует уравновешенная система пяти сил

    3. Поместив начало осей координат в точке Е и расположив их в соответствии с выбранным направлением сил горизонтально и вертикально, составим уравнения равновесия:

    4. Находим плечи AL, AD и АК

    Теперь решаем полученные уравнения.

    Из уравнения (3)


    5. Знаки «минус» у числовых значений составляющих реакции шарнира А показывают, что составляющая направлена по горизонтали влево, а — по вертикали вниз, как это показано на рис. 111,6:

    6. Находим модуль полной реакции шарнира Л и ее направление (угол на рис. 111,6):

    Из рис. 111,6 видно, что реакция шарнира А образует с брусом АВ угол () = 49°10′.

    Таким образом, реакция выступа перпендикулярна к брусу и равна н реакция шарнира направлена к брусу под углом 49°10′ и равна

    Так как направление и числовое значение полной реакции шарнирно-неподвижной опоры не зависят от первоначально предполагаемого выбора направления составляющих , то при решении подобных задач можно расположить их как угодно.

    1. Можно, например, предположить, что одна из составляющих реакции шарнира направлена вдоль бруса АВ, а вторая — перпендикулярно к нему.

    2. Изобразим при таком предположении силы, приложенные к брусу, на рис. 112, а. Расположим оси х и у как показано на том же рисунке и составим уравнения равновесия, приняв за центр моментов [для уравнения точку D:

    Теперь решим уравнения.


    Из уравнения (2)


    4. Как видно, реакция имеет такое же значение, что и в первом решении. Составляющие реакции направлены так, как показано на рис. 112, б. Используя этот рисунок, найдем модуль и направление (угол

    Как видно, результаты получаются те же; небольшое расхождение (0,7%) в значении угла, определяющем направление реакции относительно бруса АВ, объясняется приближенностью вычислений.

    Задача 18.

    Балка АВ, нагруженная как показано на рис. 114, а, удерживается в равновесии стержнями 1, 2 и 3, имеющими по
    концам шарнирные крепления. Определить реакции стержней.

    При этом

    1. На балку АВ действуют три нагрузки: в точке А— сосредоточенная сила и момент М, а на участке СВ = 6 м —равномерно

    распределенная нагрузка интенсивностью которую заменим равнодействующей приложенной в точке О — посредине участка СВ. Следовательно (рис. 114,6),

    2. Так как прямолинейные стержни при шарнирных креплениях могут только растягиваться или сжиматься, то реакции стержней направлены вдоль них. Предположим, что все стержни растянуты. Заменим их (см. рис. 114,6) реакциями

    3. Составим, как обычно, три уравнения равновесия:


    4. Из уравнения (3)


    Знак «минус» указывает, на то, что стержень 3 сжат и реакция направлена вверх.

    Из уравнения (1) выразим

    Подставим полученное значение в уравнение (2) и найдем из него .

    Таким образом, стержни 1 и 2 растянуты и их реакции стержень 3 сжат, его реакция

    Рассмотренное решение неудобно тем, что оно требует подстановки в одно из уравнений неизвестного из другого уравнения.

    Если из числа трех опорных стержней два имеют общий шарнир, то задачу можно решить иначе. Сначала определить реакцию общего шарнира, а затем, используя правило треугольника, найти реакции сходящихся у шарнира стержней.

    В рассмотренной задаче обе нагрузки действуют вертикально, а момент только стремится повернуть балку; значит нет усилий, смещающих балку в горизонтальном направлении. Поэтому аналогично тому, как указывалось в задачах 4, нагрузки могут быть уравновешены двумя реакциями, перпендикулярными к балке. А так как реакция стержня 3 перпендикулярна к балке, то и равнодействующая реакций 1 и 2 перпендикулярна к ней. На этом и основывается следующее решение.

    1. В отличие от первого решения реакции стержней 1 и 2 заменим их равнодействующей Тогда расчетная схема примет вид, показанный на рис. 115, а (штриховыми линиями показаны положения стержней 1 и 2).

    2. Составим два уравнения моментов, приняв за центры моментов точки С и D:

    3. Уравнение (1) аналогично уравнению (3) в первом решении. Решая уравнение (1), найдем, что


    Таким образом, вертикальная равнодействующая реакций и двух первых стержней равна 134 кн.

    4. Применив правило треугольника, разложим силу на составляющи (рис. 115,6), направления которых известны (реакции направлены вдоль стержней ).

    На векторе как на стороне построим треугольник abc, стороны ас и сb которого, изображающие искомые реакции стержней, соответственно параллельны стержням

    5. На основе теоремы синусов


    Следующую задачу рекомендуется решить самостоятельно.

    Справочный материал по статике

    В статике изучается равновесие тел под действием сил и свойства систем сил, необязательно находящихся в равновесии.

    Задачи статики можно условно разделить на три типа: задачи на равновесие системы сходящихся сил, т.е. сил, линии действия которых пересекаются в одной точке, задачи произвольной плоской системы сил и задачи пространственной системы сил.

    Нахождение координат центра тяжести тоже считается задачей статики. Хотя силы в этой задаче явно не присутствуют, основные формулы задачи следуют из уравнений равновесия системы параллельных сил.

    Искомыми величинами в задачах статики могут быть реакции опор, усилия в элементах конструкций, геометрические (размеры, углы) и материальные (вес, коэффициент трения) характеристики систем. В статически определимых задачах число уравнений равновесия совпадает с числом неизвестных. Именно такие задачи и будут рассмотрены в этой части.

    Для решения задач статики потребуются понятия проекции силы на ось и момента силы относительно точки и оси. Напомним, что проекция вектора силы на ось х определяется по формуле где а — угол между положительным направлением оси и вектором силы, отсчитываемый против часовой стрелки. Если угол острый, то проекция положительная, если тупой — отрицательная.

    Общее определение момента силы относительно точки О дается векторным произведением

    где — радиус-вектор точки приложения вектора силы относительно точки О. Модуль момента вычисляем по формуле

    где — угол между векторами Направление вектора момента вычисляется по правилу векторного произведения. Плечо силы относительно точки О — это кратчайшее расстояние от точки до линии действия силы;

    Вектор момента перпендикулярен плоскости, в которой располагаются силы. Поэтому в задачах статики плоской системы сил момент можно рассматривать как скалярную величину — величину проекции вектора момента на нормаль к плоскости (ось ). Индекс для сокращения записи часто опускают и отождествляют момент силы относительно точки на плоскости со скалярной величиной — Отсюда вытекает практическое правило определения момента силы относительно точки в плоских задачах статики. Для вычисления момента силы относительно точки О (рис. 1) сначала находим проекции силы на оси, а затем момент вычисляем по формуле Другой способ вычисления момента: — плечо силы относительно точки О.

    Знак определяется по правилу векторного произведения. Если сила поворачивает тело относительно центра по часовой стрелке — момент отрицательный, против часовой стрелки — положительный. На рис. 2 момент силы относительно точки О отрицательный. Если сила или линия ее действия пересекает точку, то момент силы относительно этой точки равен нулю.

    При решении задач пространственной статики (§ 4.3 — § 4.6) требуется вычислять момент силы относительно оси, или, что то же, проекцию момента силы относительно точки (1) на ось, проходящую через нее. Иногда эту величину удобнее искать как момент проекции силы на плоскость, перпендикулярную оси, относительно точки пересечения оси с плоскостью (рис. 3). Знак определяем по направлению вращения вокруг оси с точки зрения наблюдателя, находящегося на конце оси. Если вращение происходит по часовой стрелке, то момент отрицательный, против часовой стрелки — положительный.

    Момент силы относительно оси равен нулю, если сила параллельна оси или пересекает ее, т.е., если сила и ось лежат в одной плоскости.

    Кроме сил в статике рассматриваются и пары сил. Пара — .это совокупность двух равных параллельных противоположно направленных сил. Пара характеризуется моментом — суммой моментов ее сил относительно некоторой точки. Легко показать, что положение точки не существенно и на величину момента не влияет, поэтому момент пары является свободным вектором. Напомним, что вектор силы является вектором скользящим. В зависимости от знака момента пары на плоскости изображать пару будем изогнутой стрелкой Не путать эту стрелку с вектором пары! Вектор пары перпендикулярен ее плоскости.

    Решение двух задач статики в системе Maple V приведено в § 15.1, 15.2. Большинство задач статики сводится к решению систем линейных уравнений. Рутинную часть работы по составлению и решению уравнений можно поручить Maple V. Простейшая программа может выглядеть, например, так:

    Записывая уравнение на компьютере, а не на бумаге, вы достигаете сразу же нескольких целей. Во-первых, компьютер выполняет математические действия, часто весьма громоздкие. Во-вторых, уравнение легко поправить и сразу же пересчитать, если вы ошиблись при составлении уравнения и ответ не сходится. В-третьих, решение удобно оформить, распечатав его на принтере. Можно вывести график, таблицу результатов и т.д. Все эти действия можно выполнить и в других системах, в частности, в пакете AcademiaXXI.

    Плоская система сходящихся сил

    При изучении темы ПЛОСКАЯ СИСТЕМА СХОДЯЩИХСЯ СИЛ вы научитесь составлять уравнения проекций и решать задачи равновесия плоских стержневых систем методом вырезания узлов. Этот метод лежит в основе компьютерной программы расчета ферм (§15.1).

    Простая стержневая система

    Постановка задачи. Плоская шарнирно-стержневая конструкция закреплена на неподвижном основании и нагружена в шарнирах силами. Найти усилия в стержнях.

    Рассматриваем равновесие внутренних шарниров системы, не соединенных с неподвижным основанием. Такие шарниры будем называть узлами. Действие каждого стержня заменяем его реакцией — силой, направленной из узла к стержню. Усилие — это проекция реакции стержня на внешнюю нормаль к сечению. Если в результате решения задачи реакция стержня, приложенная таким образом к узлу, оказывается отрицательной, то стержень сжат, в противном случае стержень растянут.

    • 1. Вырезаем узел, соединенный только с двумя стержнями. Действие стержней заменяем их реакциями.
    • 2. Для полученной системы сходящихся сил составляем уравнения равновесия в проекциях на выбранные для этого узла оси.
    • 3. Решаем систему двух линейных уравнений и находим искомые усилия.
    • 4. Вырезаем очередной узел системы, тот, к которому подходят не более двух стержней с неизвестными усилиями. Составляем и решаем уравнения равновесия в проекциях на оси, выбранные для этого

    Простая стержневая система:

    узла. Этот пункт плана выполняем несколько раз для всех узлов до нахождения всех усилий.

    • 5. Для проверки решения мысленно отделяем конструкцию от основания, заменяя действие рассеченных стержней найденными реакциями. Проверяем выполнение условий равновесия полученной системы сил.

    Замечание 1. Существуют фермы , у которых к каждому узлу присоединены более двух стержней. Например, на рис. 4 изображена конструкция (сетчатая ферма В.Г.Шухова), к каждому узлу которой подходит по три стержня. Диагональные стержни расположены в разных плоскостях и не пересекаются.

    Здесь нельзя определять усилия по предложенной схеме, переходя от одного узла к другому, так как нет узла, с которого можно начать расчет. В этом случае сначала составляются уравнения равновесия отдельных узлов, а потом совместно решается система полученных уравнений. Систему можно решать любым известным способом.

    Замечании 2. Для упрощения уравнений равновесия одну из осей координат можно направить вдоль стержня с неизвестным усилием. Для каждого узла можно выбрать свою систему координат.

    Замечание 3. Углы между осями и векторами усилий легче определять, если проводить через узлы вспомогательные вертикальные или горизонтальные прямые.

    Замечание 4. Усилия в стержнях можно найти с помощью системы Maple V (Программа 1, с. 3-50).

    *)Шарнирно-стержневая конструкция, нагруженная в шарнирах силами, называется фермой. Весом стержней фермы и трением в шарнирах пренебрегают.

    Пример. Плоская шарнирно-стержневая конструкция закреплена на неподвижном основании шарнирами Е, D, С и нагружена в шарнире А горизонтальной силой Р = 100 кН (рис. 5). Даны утлы: Найти усилия в стержнях.

    Конструкция состоит из шести стержней, соединенных тремя шарнирами (узлами). Узлы фермы находятся в равновесии. Для каждого узла А, В, F составляем по два уравнения равновесия в проекциях на выбранные оси. Из шести уравнений находим шесть искомых усилий.

    1. Решение задачи начинаем с рассмотрения узла А, так как этот узел соединен только с двумя стержнями А В и AF. При вырезании узла действие каждого стержня заменяем силой, направленной из шарнира к стержню (рис. 6).

    2. Составляем уравнения равновесия. Для упрощения уравнений ось направляем по стержню АВ. Получаем

    где — проекции силы на ось х, a — проекции силы на ось

    3.Решаем уравнения. Из первого уравнения системы находим усилие из второго — усилие

    4. Рассматриваем узел F. К нему подходят три стержня (рис. 7).

    Усилие в одном из них уже известно Усилия в двух других находим из уравнений для проекций:

    Находим

    Составляем уравнения равновесия узла В в проекциях на оси, направленные по стержням ВС и BD (рис. 8):

    Решая уравнения, получаем:

    5. Проверка. Рассматриваем равновесие конструкции в целом.

    Горизонтальным сечением отсекаем ферму от основания. Действия стержней заменяем силами, которые направляем, как и раньше, по внешним нормалям к сечениям стержней, т.е. вниз (рис. 9).

    Система сил, действующих на ферму, не является сходящейся. Для такой системы справедливы три уравнения равновесия, одно из которых — уравнение моментов. Составление уравнения моментов — тема задач статики произвольной плоской или пространственной системы сил (§2.1 — 3.2). Для того, чтобы не выходить за пределы темы поставленной задачи, в решении которой используются только уравнения проекций, составим два уравнения проекций на оси всех сил, действующих на ферму целиком:

    Суммы равны нулю. Это подтверждает правильность решения. Результаты расчетов в кН заносим в таблицу

    51.76-73.2173.21-26.7936.60-63.40

    Равновесие цепи

    Постановка задачи. Определить положение равновесия плоского шарнирно-стержневого механизма, состоящего из последовательно соединенных невесомых стержней. Механизм расположен в вертикальной плоскости. В крайних точках механизм шарнирно закреплен на неподвижном основании. Средние шарниры нагружены силами. Найти усилия в стержнях.

    Особенностью задачи является необычный для статики объект исследования — механизм, имеющий возможность двигаться. При определенном соотношении нагрузок и геометрических параметров механизм принимает положение равновесия. В качестве искомой величины может быть угол или какая-либо другая геометрическая характеристика конструкции. План решения

    • 1. Записываем уравнения равновесия узлов системы в проекциях.
    • 2. Решаем полученную систему уравнений. Определяем усилия в стержнях и искомый угол.
    • 3. Проверяем равновесие конструкции в целом, освобождая ее от внешних связей. Проверочным уравнением может быть уравнение проекций на какую-либо ось.

    Задача 19.

    Определить положение равновесия плоского симметричного шарнирно-стержневого механизма. Концы А и Е шарнирно закреплены на неподвижном основании. Три внутренних шарнира В, С и D нагружены одинаковой вертикальной нагрузкой Q.
    В положении равновесия — 60°. Определить угол и усилия в стержнях (рис. 10). Весом стержней пренебречь.

    Конструкция, данная в условии задачи, представляет собой механизм, находящийся в равновесии только при некоторых определенных нагрузках. При изменении направлений и величин нагрузок меняется и конфигурация конструкции. Одной из неизвестных величин задачи (помимо усилий в стержнях) является угол . Для решения задачи используем метод вырезания узлов.

    1. Записываем уравнения равновесия узлов системы. Составим уравнения равновесия узла С (рис.11):

    Конструкция симметрична, поэтому уравнения равновесия узлов В и D запишутся одинаково. Рассмотрим равновесие узла В (рис.12).

    Для упрощения уравнений направим ось у по стержню АВ, ось х — перпендикулярно АВ. Тогда, уравнение равновесия в проекции на ось х содержит только одну неизвестную величину:

    2. Решаем систему уравнений (1-4). Из (1) получаем, что Это равенство объясняется симметрией конструкции и симметрией нагрузок. Из (2) и (4) с учетом полученного равенства находим

    Выражаем из (5) и подставляем в (3):

    Так как то после сокращения на получаем уравнение для

    или Из (5) получаем усилие Стержень ВС сжат. Из (6) находим усилие

    В силу симметрии задачи Результаты расчетов заносим в таблицу:

    3. Проверка. Рассмотрим равновесие всей конструкции в целом

    Отсекая стержни от основания, заменим их действие реакциями, направленными по внешним нормалям к сечениям стержней, т.е. вниз (рис. 13). Уравнение проекций на ось х составлять не имеет смысла — в силу симметрии оно лишь подтвердит, что Проверяем равенство нулю суммы проекций всех сил на вертикаль:Задача решена верно.

    Теорема о трех силах

    Постановка задачи. Тело находится в равновесии под действием трех сил, одна из которых известна, у другой известно только направление, а у третьей не известны ни величина, ни направление. Используя теорему о трех силах, найти неизвестные силы.

    В теореме о трех силах утверждается, что если на тело, находящееся в равновесии, действуют три непараллельные силы (включая реакции опор), то они лежат в одной плоскости, и линии их действия пересекаются в одной точке.

    • 1. Найдем точку пересечения линий действия двух сил, направления которых известны. Через эту точку должна пройти и линия действия третьей силы.
    • 2. Имея направления векторов трех сил, строим из них силовой треугольник. Начало одного вектора является концом другого. Если тело находится в равновесии, то сумма векторов сил, действующих на него, равна нулю. Следовательно, треугольник сил должен быть замкнут.
    • 3. Из условия замкнутости треугольника по направлению заданной силы определяем направление обхода треугольника и, следовательно, направления искомых сил.
    • 4. Находим стороны силового треугольника — искомые силы.

    Задача 20.

    Горизонтальный невесомый стержень А В находится в равновесии под действием трех сил, одна из которых вертикальная сила F = 5 кН (рис. 14), другая — реакция опорного стержня CD, а третья — реакция неподвижного шарнира А. Используя теорему о трех силах, найти неизвестные реакции опор.

    1.3. Теорема о трех силах

    1. Найдем точку пересечения линий действия двух сил, направления которых известны. Определим направление линии действия третьей силы.

    На стержень АВ действуют три силы: заданная сила реакция шарнира А и реакция стержня CD. При этом линия действия вектора известна. Она совпадает со стержнем CD, так как стержень нагружен только двумя силами в точках С и D (вес стержня не учитывается). Согласно аксиоме статики эти силы равны по величине и направлены вдоль CD в разные стороны. Направление реакции шарнира А определяем по теореме о трех силах. Линии действия сил пересекаются в точке О (рис. 15). Следовательно, АО — линия действия силы Известны только линии действия сил поэтому векторы на рис. 15 не изображаем, пока из силового треугольника не узнаем их направления.


    2. Строим силовой треугольник. Сумма векторов сил, находящихся в равновесии, равна нулю, следовательно, треугольник, составленный из должен быть замкнут.

    Треугольник строим, начиная с известной силы (рис. 16). Через начало и конец вектора проводим прямые, параллельные направлениям

    3.Из условия замкнутости треугольника по направлению внешней силы определяем направление обхода треугольника и, следовательно, направления реакций опор.

    Замкнутость треугольника сил означает, что начало одной силы совпадает с концом другой. Отсюда определяем направление обхода треугольника, которое может быть различным в зависимости от способа построения силового треугольника (рис. 17 — против часовой стрелки, рис. 18 — по часовой стрелке). Направления и величины сил в обоих случаях одни и те же.

    Изобразим реакции с учетом найденных направлений (рис. 19).

    4. Определяем длины сторон силового треугольника — величины реакций опор. Найти стороны треугольника сил означает решить задачу. В нашем случае известны углы (по построению) и сторона F треугольника. Две другие стороны находятся по теореме синусов.

    Можно поступить иначе, используя свойства подобия. На рис. 15 найдем треугольник подобный силовому. В ряде случаев этот треугольник очевиден. В общем же, для получения такого треугольника надо выполнить дополнительные построения: провести линии, проходящие через характерные точки (шарниры, точки приложения сил и т.п.), параллельно сторонам силового треугольника. Проведем, например, вертикаль Образуется треугольник подобный силовому (рис. 15, 17). Подобие следует из условия параллельности сторон треугольников.

    Найдем стороны треугольника

    Из подобия имеем соотношения

    Отсюда вычисляем длины:

    1.3. Теорема о трех силах

    Из условия подобия треугольника сил и следует, что

    Из этих пропорций находим искомые величины:

    Предупреждение типичных ошибок

    1. Размеры на чертеже сил, приложенных к телу (рис.15), измеряются в единицах длины (м, см), а на силовом треугольнике (рис. 17, 18) в единицах сил Не надо принимать линейные расстояния АО, СО и ВО за величины соответствующих сил.
    2. Реакция гладкого основания перпендикулярна поверхности основания. Реакция гладкой поверхности тела о неподвижную опору перпендикулярна поверхности тела.
    3. В данной задаче должно быть только три силы. Лишние силы возникают, если прикладывать вес тела там, где его нет, или если реакцию в шарнире А раскладывать на составляющие.
    Рекомендую подробно изучить предмет:
    • Теоретическая механика
    Ещё лекции с примерами решения и объяснением:
    • Трение
    • Пространственная система сил
    • Центр тяжести
    • Кинематика точки
    • Моменты силы относительно точки и оси
    • Теория пар сил
    • Приведение системы сил к простейшей системе
    • Условия равновесия системы сил

    При копировании любых материалов с сайта evkova.org обязательна активная ссылка на сайт www.evkova.org

    Сайт создан коллективом преподавателей на некоммерческой основе для дополнительного образования молодежи

    Сайт пишется, поддерживается и управляется коллективом преподавателей

    Whatsapp и логотип whatsapp являются товарными знаками корпорации WhatsApp LLC.

    Cайт носит информационный характер и ни при каких условиях не является публичной офертой, которая определяется положениями статьи 437 Гражданского кодекса РФ. Анна Евкова не оказывает никаких услуг.


    источники:

    http://electrichelp.ru/teoreticheskaya-mexanika-v-pomoshh-studentu/

    http://www.evkova.org/ploskaya-sistema-sil-v-teoreticheskoj-mehanike